USMLE Step 3: Part 2

Pataasin ang iyong marka sa homework at exams ngayon gamit ang Quizwiz!

All of the following are a/w reduced lifetime risk of developing Breast Cancer, except: A: absence of a hx of maternal nursing B: first full term pregnancy before age 18 C: menarche after age 15 D: natural menopause before 42 YO E: surgical menopause before 42 YO

A Approximately 80-90% of the variation in BCA frequency in different countries can be attributed to differences in menarche, first pregnancy, and menopause. Women who experience menarche at age 16 have 40-50% the risk of BCA of women who experience menarche at 12 YO. Menopause, surgical or natural, occurring 10 years before the median age of 52 reduces the risk of BCA by 35%. Women who have the first full term pregnancy by age 18 have a 30-40% reduced risk of BCA compared with nulliparous women. These date taken together suggest that a substantial portion of the risk of BCA is related directly to the length of menstrual cycle, particularly the fraction occurring before the first full term pregnancy. Independent of these RF, the duration of maternal nursing is a/w a reduction in BCA risk.

68 YO M seeks evaluation for fatigue, wt loss, and early satiety that has been present for about 4 months. On PE, spleen is noted to be markedly enlarged. It is firm to the touch and crosses the midline. The lower edge of the spleen reaches to the pelvis. Hgb 11, HCT 34, WBC 6.2, PLT 220; DDX 75% PMNs, 8% myelocytes, 4% metamyelocytes, 8% lymphocytes, 3% monocytes, and 2% eosinophils. The PBS shows teardrop cells, nucleated RBCs, and immature granulocytes. RF is positive. BM biopsy is attempted, but no cells are aspirated. No evidence of leukemia or lymphoma is found. What is the cause of the splenomegaly? A: chronic idiopathic myelofibrosis B: CML C: rheumatoid arthritis D: SLE E: TB

A Chronic idiopathic myelofibrosis (IMF) is the last common myeloproliferative disorder and is considered a diagnosis of exclusion after other causes are r/o. The typical patients is in their 6th decade and the disorder is asymptomatic in many patients. Fever, fatigue, NS, and wt loss may occur whereas these ssx are rare in other myeloproliferative. However, no ssx are specific for the diagnosis. Often, marked splenomegaly is present and may extend across the midline and to the pelvic brim. PBS shows typical findings of teardrop-shaped RBCs, nucleated RBCs, myelocytes, and metamyelocytes that are indicative of extra medullary hematopoiesis. Anemia is usually mild, and PLT and WBC counts are often wnl. BM aspirate is frequently unsuccessful because the extent of fibrosis makes aspiration impossible. When a biopsy is performed, it demonstrates hypo cellular marrow with trilineage hyperplasia and increased # of megakaryocytes with large, dysplastic nuclei. Interestingly, IMF often have associated autoantibodies, including RF, ANA, or a positive Coomb's test. To dx someone as having IMF, it must be shown they have no other proliferative or heme malignancy that is the cause. The MC disorders that present similarly are PV and CML. Nonmalignant disorders like HIV, hyperPTH, renal osteodystrophy, SLE, TB and marrow replacement in other cancers such as prostate or breast. In this patient, there are no other identifiable RF, thus it is dx as IMF

76 YO M presents to an urgent care center with pain in the L LE for 4 days. He also describes swelling in the L ankle, which has been difficult to ambulate. He is an active smoker and has a PMH remarkable for GERD, prior DVT 9 months ago (resolved), and well controlled HTN. PE shows 2+ edema in L ankle. A D-dimer is ordered and elevated. Which makes D-dimer less predictive of DVT in this patient? A: Age >70 B: hx active tobacco use C: lack of suggestive clinical symptoms D: negative Homan's sign on PE E: previous DVT in the past year

A D-dimer is a degradation production of cross-linked fibrin and is elevated in conditions of ongoing thrombosis. Low [ ] of d-Dimer are considered to indicate the absence of a thrombosis. Patients > 70 will often have elevated D-dimers in the absence of thrombosis, making this test less predictive of acute disease. Clinical symptoms are often not present in patients with DVT and do not affect interpretation of D-dimer. Tobacco use, while frequently considered a risk factor for DVT, and previous DVT should not affect the predictive value of D-dimer. Homan's sign (calf pain elicited by dorsiflexion of the foot), is not predictive of DVT and is unrelated to D-dimer

Which of the following statements regarding incidence of and RF for hepatocellulr carcinoma is true? A: a chemical toxin produced by Aspergillus spp, aflatoxin B, has a strong a/w development of HCC and can be found in stored grains in hot, humid places B: in the US, the incidence of HCC is decreasing C: non-ETOH steatohepatitis is not a/w increased risk for HCC D: fewer than 5% of individuals diagnosed with HCC in the US do not have underlying cirrhosis E: the risk of HCC in individuals with HCV is about 50%

A HCC is one of the MC cancers worldwide with the highest incidence in SE Asia and sub-saharan Africa. However, the incidence of HCC in the US is rapidly increasing and is thought to be related to increased number of individuals infected with HCV. At present, an estimated 4 million people are infected with HCV, of whom 10% have cirrhosis. Of those who develop cirrhosis from HCV, ~5% will develop HCC. Other common RF for HCC include cirrhosis from any cause, chronic HBV or HCV, alcoholism, non-ETOH steatohepatitis, aflatoxin B exposure, and primary biliary cirrhosis. Aflatoxin B is a mycotoxin produced by Aspergillus spp that is found in stored grains in hot, humid places. It is the best studied and most potent naturally occurring carcinogen a/w HCC. In the US, 20% of individuals diagnosed with HCC do not have cirrhosis. In these individuals, the etiology of HCC is unknown and the natural hx is not well defined.

Which of these is true? A: factor VIII def is characterized clinically by bleeding into soft tissues, ms, and weight bearing joints B: congenital factor VIII def is inherited in an AR fashion C: factor VIII def results in prolongation of the PT D: factor VIII complexes with Hageman factor, allowing for a longer half life E: factor VIII has a half life of nearly 24 h

A Hemophilia A results from an inherited def of factor VIII. The gene for factor VIII is on the X chromosome. Therefore, its X-linked inheritance pattern results in approx 1 in 10,000 males being born with some level of dysfunction. Clinically, it is characterized by bleeding into soft tissues, ms, and weight-bearing joints. Symptomatic patients usually have levels below 5%. Bleeding occurs hours or days after an injury and can involved any organ. Factor VIII is involved in the intrinsic pathway of coagulation. Therefore, def usually results in abnormalities of the aPTT. Factor VIII has a very short half life of 8-12 Horus. Therefore, repeat transfusions of plasma, cryoppt, or purified factor VIII must be given bid. Factor VIII complexes with vWF, not Hageman factor.

Which of these statements regarding polycythemia vera is correct? A: an elevated plasma EPO excludes the diagnosis B: transformation to acute leukemia is common C: thrombocytosis correlates strongly with thrombotic risk D: ASA should be prescribed to all these patients to reduce thrombotic risk E: phlebotomy is used only after hydroxyurea and IFN have been tried

A PV is a clonal disorder that involves a multipotent hematopoietic progenitor cell. Clinically, it is characterized by a proliferation of RBCs, granulocytes, and PLTs. The precise etiology is unknown. Erythropoiesis is regulated by the hormone EPO. Hypoxia is the physiologic stimulus that increases the number of cells that produce EPO. EPO may be elevated in patients with hormone-secreting tumors. Levels are usually "normal" in patients with hypoxic erythrocytosis. In PV, however, because erythrocytosis occurs independently of EPO, levels of the hormone are usually low. Therefore, an elevated level is not c/w the diagnosis. Polycythemia is a chronic, indolent disease with a low rate of transmission to acute leukemia, especially in the absence of treatment with radiation or hydroxyurea. Thrombotic cx are the main risk for PV and correlate with the erythrocytosis. Thrombocytosis, although sometimes prominent, does not correlate with the risk of thrombotic cx. Salicylates are useful in treating erythromelalgia but are not indicated in asymptomatic patients. There is no evidence that thrombotic risk is significantly lowered with their use in patients who HCT are appropriately controlled with phlebotomy. Phlebotomy is the mainstay of treatment. Induction of a state of iron deficiency is critical to prevent a reexpansion of the RBC mass. Chemotherapeutics and other agents are useful in cases of ssx splenomegaly. Their use is limited by AE, and there is a risk of leukemogenesis with hydroxyurea.

which of the following should prompt investigation for HNPCC screening in a 32 YO male? A: father, paternal aunt, and paternal cousin with CRC at 54, 68, and 37 years, respectively B: innumerable polyps visualized on routine colonoscopy C: mucocutaneous pigmentation D: new diagnosis of UC E: none of these

A a strong FamHx of CRC should prompt consideration for HNPCC, or Lynch Syndrome, particularly if diffuse polyposis is NOT noted on colonoscopy. HNPCC is characterized by: (1) 3 or more relatives with histologically proven CRC, one of whom is 1st degree relative, and of the other two, at least 1 with the diagnosis < 50 YO; (2) CRC in at least 2 generations the disease is AD and is a/w other tumors, including in the endometrium and ovary. The proximal colon is most frequently involved, and cancer occurs with a median age of 50 YO, 15 years earlier than in sporadic CRC. Patients with HNPCC are recommended to receive biennial colonoscopy and pelvic US beginning at age 25 YO. Innumerable polyps suggest the presence of one of the AD polyposis syndromes, many of which have high malignant potential. These include FAP, Gardner's syndrome (a/w osteomas, fibromas, epidermoid cyts), or Turcot's syndrome (a/w brain cancer). Peutz-Jegher's syndrome is a/w mucocutaneous pigmentation and hamartomas. Tumors may develop in the ovary, breast, pancreas, and endometrium; however, malignant tumors are uncommon. UC is strongly a/w development of CRC, but it is unusual for CRC to be the presenting finding in UC. Patients are generally symptomatic from their IBD long before cancer risk develops.

all of the following types of cancer commonly mets to the CNS, except: A: ovarian B: breast C: hypernephroma D: melanoma E: ALL

A about 25% of patients with cancer die from intracranial mets. SSX may relate to the parenchymal or leptomeningeal involvement. The ssx of mets brain tumors are similar to other intracranial expanding lesions: HA, n/v, behavioral changes, seizures, and focal neurologic deficits. 3-8% of patients with cancer develop a tumor in the leptomeninges. These patients typically present with multifocal neurologic ssx. Signs include CN palsies, extremity weakness, paresthesias, and loss of DTRs. CT and MRI are useful in dx of intraparenchymal lesions. The treatment of choice is radiotherapy. Solitary lesions in selected patients may be resected to achieve improved disease-free survival. The diagnosis of leptomeningeal disease is made by demonstrating tumors in the CSF. Each attempt has limited sensitivity, and so patients with clinical features suggestive of leptomeningeal disease should undergo (3) serial CSF samplings. Neoplastic meningitis usually occurs in the setting of uncontrolled cancer outside the CNS. Therefore, the prognosis is typically dismal, with a median survival between 10-12 weeks.

You are investigating the cause for a patient's anemia. He is 50 YO M who was found to have a HCT 25 on routine evaluation. His HCT was 47, 1 year ago. MCV is 80, MCHC is 25, mean corpuscular hemoglobin is 25. Reticulocyte count is 5%. Review of the PBS shows marked numbers of polychromatophilic microcytes. Ferritin is 340. What is the cause? A: defective erythroid marrow proliferation B: extravascular hemolysis C: intravascular hemolysis D: iron-deficiency anemia E: occult GIT bleed

A an accurate reticulocyte count is a critical component of lab workup for anemia. There are two corrections that need to be made to the count when it is being used to estimate the marrow's response to anemia. (1) The first correction adjusts the count for the number of circulating RBCs (i.e., the % of reticulocytes may be increased although the absolute number is unchanged). The absolute reticulocyte count = reticulocyte count * (HCT/expected HCT) (2) Secondly, when there is evidence of prematurely released reticulocytes on blood smear (polychromatophilia), a prolonged maturation in the serum may cause a falsely high estimate of daily RBC production. Correction is achieved by dividing by a "maturation time correction", usually 2 if the HCT is between 25-35%. In this example, the reticulocyte production index is 5*(25/45)/2, or 1.4. If a reticulocyte production index is <2 in the face of anemia, a defect in the erythroid marrow proliferation must be present. GIT bleeds should be considered in this demographic; however, a lower reticulocyte count with normal iron stores argue strongly for a defect in the erythrocyte proliferation. A ferritin > 200 indicates that there are some iron stores present. Clues for extravascular hemolysis include an elevated LDH, spherocytes on PBS, and HSM. Intravascular hemolysis (DIC, mechanical heart valve, TTP) will show schistocytes on peripheral smear.

64 YO M with Child-Pugh Class B cirrhosis presents to GI c/o weight loss and feeling of ABD fullness. He was diagnosed with HCV cirrhosis 5 years previously. It is thought that the patient developed HCV s/p blood transfusion 20 years ago after a MVA. His initial presentation was cirrhosis with volume overload and ascites. He has been successfully managed with Na restriction, spironolactone, and furosemide. He has no other significant PMH. On exam today, his liver is enlarged and firm. No ascites is present. A helical CT of the abdomen shows a single tumor in the R Lobe of the liver measuring 4cm in diameter. The location of the mass is near the main portal pedicles. There is no evidence of vascular invasion or mets. The AFP is 384. Bx of the mass is diagnostic for HCC. What is the best treatment? A: liver transplant B: radio frequency ablation C: resection of the R lobe D: systemic CTX E: transarterial chemoembolization

A currently, HCC can be staged using a variety of staging systems. The TNM has been replaced by either Okuda or Cancer of Liver Italian (CLIP) system because they include the presence of cirrhosis as part of stage. This patient would have stage II disease by TNM because he has a single tumor >2cm but w/o evidence of vascular invasion. By CLIP, he would be classified as CLIP 1 because of his Child Pugh Class B cirrhosis. Primary surgical resection of the mass is reserved for individuals with stage I or II HCC or Clip Stage 0. However, because of the high rate of liver failure and mortality s/p resection in Class B or C cirrhosis, these individuals are not good candidates for resection. Othrotopic liver transplant is the treatment in individuals with Stage I or II disease and cirrhosis. They can be referred if there is a single mass <5cm or three masses <3cm and no vascular invasion. RF ablation uses heat to cause necrosis of ~7cm zone in a nonspecific manner. This technique can be used efficiently in a single lesion that are 3-4 cm in size. However, tumors located near the main portal pedicles can lead to bile duct injury and obstruction. Percutaneous ethanol injection (not listed) results in necrosis of the injected area and requires multiple injections. The max size of the tumor that can be treated this way is 3cm. Transarterial chemoembolization is a form of regional CTX in which a variety of CTX agents are injected into the hepatic artery. Two randomized trials have shown a survival advantage for transarterial chemoembo in a highly select group of patients. The technique is recommended for individuals who are not candidates for transplant, including those with multiple comorbs, >4 lesions, LN mets, tumors >5cm, and gross vascular invasion. Systemic CTX has no effect on survival and has a <25% response rate. It is not recommended for most individuals with HCC. Sorafenib is a new agent that increases median survival from 6 months to 9 months in patients with advanced disease.

26 YO F who is 4 months pregnant is seen for standard evaluation. She reports feeling well with decreased nausea over the past 1 month. The PE is wnl except for the presence of 1.5-cm hard nodule in the upper outer quadrant of the R breast. She does not recall the nodule being present previously and has not performed self exam since becoming pregnant. Which is the most appropriate? A: aspiration of the nodule B: mammogram after delivery C: Rx of po Progesterone D: recommendation of genetic testing for BRCA E: repeat PE after delivery

A during pregnancy the breast grows under the influence of estrogen, progesterone, and PRL, and human placental lactose. However, the presence of a dominant breast nodule/mass during pregnancy should never be attributed to hormones. Breast Ca develops in 1:3000 to 4000 pregnancies. The prognosis for BCA by stage is no different in pregnant compared to non pregnant women. Nevertheless, pregnant women are often diagnosed with more advanced disease because of delay. Pregnant patients with persistent lumps in the breast should receive prompt diagnostic evaluation.

all of these labs are c/w an intravascular hemolytic anemia, except: A: increased haptoglobin B: increased LDH C: increased reticulocyte count D: increased unconjugated bilirubin E: increased urine hemosiderin

A haptoglobin is an alpha-globulin normally present in serum. It binds specifically to the globin portion of the hemoglobin, and the complex is cleared by the monocytes. Haptoglobin is reduced in all hemolytic anemias as it binds free hemoglobin. It can also be reduced in cirrhosis and so is not diagnostic of hemolysis outside of the correct clinical context. assuming a normal marrow and iron stores, the reticulocyte count will be elevated as well to try to compensate for the increased RBC destruction of hemolysis. Release of intracellular contents from the RBC (including Hgb and LDH) induces heme metabolism, producing unconjugated bilirubin. If the haptoglobin system becomes overwhelmed, the kidney will filter free Hgb and reabsorb it in the PCT for storage of iron by ferritin and hemosiderin. Hemosiderin in the urine is a marker of filtered hemoglobin by the kidneys. In massive hemolysis, free Hgb may be excreted in the urine

Which of the following statements about lead bias occurrence is true? A: a test does not influence the natural history of the disease; patients are merely diagnosed at an earlier date B: slow growing, less aggressive cancers are detected during screening; aggressive cancers are not detected by screening, due to death C: screening identifies abnormalities that would never have caused a problem in the person's lifetime D: the screened population differs significantly from the general population in that they are healthier E: a test detects disease at an earlier and more curable stage of the disease

A lead time bias, length time bias, selection bias, over diagnosis bias, and avoidance bias can make a screening test appear to improve outcomes when it does not. When lead-time bias occurs, survival appears increased, but life is not truly prolonged. The test only lengthens the time that the patient, physician, or investigator are aware of the disease. When length time bias occurs, aggressive cancers are not detected during screening, presumably because of higher mortality from these cancers and the length of the screening interval. Selection bias can occur when the test population is either healthier or at higher risk for developing the condition than the general population. Overdiagnosis bias, such as with some indolent forms of prostate cancer, detects conditions that will never cause significant morb/mort during a person's lifetime. The goal of screening is to detect disease at an earlier and more curable stage.

All of the following statements regarding gastric carcinoma are true, except: A: linitis plastica is an infiltrative form of gastric lymphoma with no defined margins that carries a poorer prognosis than intestinal-type lesions B: reduction of tumor bulk with surgery is the best therapeutic option for gastric adenocarcinoma, if surgically feasible C: long term ingestion of high concentrations of nitrates in dried, smoked, or salted foods is a/w gastric cancer rates D: the presence of palpable, firm, periumbilical nodules is a poor prognostic sign E: ulcerative lesions in the distal stomach should always undergo brush sampling with biopsy to r/o adenocarcinoma

A the rates of gastric cancer have declined significantly over the past 75 years. Nevertheless, there were >20,000 new cases in the US with >10,000 deaths in 2007. Gastric cancer is still high in Japan, China, Chile, and Ireland. Epidemiology shows that a higher prevalence in lower SES groups and the maintenance of risk for individuals but not offspring migrating from high to low risk environments suggests an environmental exposure early in life is a risk. Risk is a/w ingestion of high nitrite foods. The nitrites may be converted to carcinogens by bacteria in partially decayed food. Chronic gastritis and achlorhydria due to H pylori gastric infection may contribute to the risk. The effect of eradication of H pylori, improved food preservation, and widespread refrigeration may all be contributing to its decline. The MC type is adenocarcinoma. The majority of which occur in the distal stomach and appear ulcerative on contrast radiography and endoscopy. Therefore, all gastric ulcers warrant a biopsy and brushing for early detection of adenocarcinoma in the stomach. Such early lesions have the highest likelihood for surgical cure. Some 13% of gastric adenocarcinomas are diffuse-type, involve most of the stomach, and are referred to as linitis plastic based on poor distensibility of the stomach. Prognosis for diffuse carcinoma is worse than for intestinal type, and this is more common in the young. There is not a good association between this type of gastric cancer and H pylori. For all adenocarcinoma, the presence of palpable periumbilical nodes, or Sister Mary Joseph's nodes, implies mets and confers a poor prognosis. Surgery should be considered first line therapy for cure, or palliation/debulking if the patient is a surgery candidate. It is critical to ddx adenocarcinoma from gastric lymphomas as lymphoma carries a much better prognosis, with H pylori eradication causing regression in 75% of cases. Antimicrobial therapy should be considered before surgery, XRT, or CTX in gastric lymphoma. Surgery, usually with CTX, may be curative in 40-60% of patients with resistant or high grade lymphoma

73 YO M admitted with 3 weeks of malaise and fevers. His PMH is notable for HTN controlled with a thiazide. He smokes 1 ppd and works as an attorney. His PE is notable for a new systolic heart murmur heard best in mitral region. His lab exam is notable for mild anemia, an elevated WBC, and occasional RBC on clean catch urine. Blood cx grow Streptococcus bovis and ECHO shows a <1cm vegetation on the mitral valve. What additional evaluation is indicated? A: colonoscopy B: head CT C: pulmonary embolism protocol CT scan D: renal biopsy E: toxicology screen

A this patient has Strep bovis endocarditis. For unknown reasons, individuals who develop endocarditis or septicemia from this fecal organism have a high frequency of occult CRC. Upper GI tumors have been described as well. All patients with this type of endocarditis should receive colonoscopy after stabilization. Tobacco use has been linked to the development of colorectal adenomas, particularly after >35 years of tobacco use, again for unknown reasons. Patients with illicit drug use (diagnosed by toxicology) are at risk for endocarditis due to Staph aureus. A head CT looking for embolic lesions is not necessary in the absence of physical findings or large vegetations that are prone to embolism. Patient with endocarditis often have renal abnormalities, including microscopic hematuria from immune complex deposition, but a renal biopsy to evaluate for glomerulonephritis is not indicated in the presence of documented endocarditis. A pulmonary embolus, while certainly possible during hospitalization, would not be a/w acute presentation of strep bovis endocarditis.

48 YO F admitted with anemia and thrombocytopenia after c/o profound fatigue. Her initial Hgb 8.5, HCT 25.7, and PLT 42. Her WBC 9.5, but 8% blasts are noted on PBS. Chromosomal analysis shows a reciprocal translocation of chromosome 15 and 17, t(15;17), and a diagnosis of acute promyelocytic leukemia is made. The induction regimen of this patient should include: A: all-trans-retinoic acid (ATRA, or triretinoin) B: arsenic C: cyclophosphamide, daunorubicin, vinblastine, and prednisone D: rituximab E: whole body irradiation After the patient is started on the appropriate treatment, she develops acute onset of SOB, fever, and CP. Her CXR shows b/l alveolar infiltrates and moderate b/l pleural effusions. WBC is now 22.3, with PMNs 78%, bands 15%, and lymphocytes 7%. She has bronchoscopy with lavage that shows no bacteria, fungi, or viral organisms. What is the most likely diagnosis?

A treatment of aPML is an interesting example of how understanding the function of the protein produced by the genetic abnormality can be utilized to develop a treatment for the disease. The translocation of the long arms of 15 and 17, results in the production of a chimeric protein called PML/retinoic acid receptor alpha (Rara). The PML-Rara fusion protein suppresses gene transcription and arrests ddx of the cells in an immature state leading to PML. Pharm doses of the ligand of the RARA-receptor, ATRA, stimulate the cells to resume ddx. With use of ATRA, the leukemic cells ddx to mature PMNs and undergo subsequent apoptosis. This leads to treatment and remission of PML w/o causing myelosuppression that is common to other CTX used for treatment of leukemia. While ATRA alone can yield heme remission of PML, it is often combined with traditional CTX in order to generate a cytogenetic remission as well. Since the introduction of ATRA, complete remission and survival have further improved to about 75-80% at 5 years. The primary AE is the development of retinoid acid syndrome. The onset of this from ATRA is usually within the first 3 weeks of treatment. Typical ssx are CP, fever, and SOB. Hypoxia is common, and CXR shows diffuse alveolar infiltrates with pleural effusions. Pericardial effusions may also occur. The cause of the syndrome is possibly related to the adhesion is possibly related to the adhesion of the ddx cells to the pulmonary endothelium or the release of cytokines by these cells to cause vascular leak. Mortality is about 10%. High dose GC therapy is usually effective. Arsenic is currently indicated for the treatment of relapsed PML and is effective in up to 85% of individuals who are refractory to ATRA. Ongoing trials are attempting to determine if combination therapy of ATRA and arsenic may further improve outcomes. Cyclophophamide, daunorubicin, vinblastine, and prednisone are the CHOP therapy, and indicated for the treatment of B lymphomas. Rituximab is MC used for B cell NHL, and currently under investigation for CLL and refractory AID, including SLE and RA. Rituxin is a monoclonal antibody directed at CD20 cell surface of B cells. Neither of these drugs are used in myeloid leukemias. Whole body irradiation is used primarily before BMT to ensure complete eradication of cancerous leukemia cells in the BM

72 YO M with COPD and stable CAD presents to the ED with several days of worsening productive cough, fevers, malaise, and diffuse ms ache. A CXR shows a new lobar infiltrate. Labs show WBC 12.1, with neutrophil predominance of 86% and 8% bands. He is dx with CA-PNA, and abx are initiated. Under normal, or "nonstress" conditions, what % of the total body neutrophils are present in circulation? A: 2% B: 10% C: 25% D: 40% E: 90%

A under normal or nonstress, roughly 90% of the neutrophil pool is in the BM, 2-3% in the circulation, and the remainder in the tissues. The circulating pool includes the freely flowing cells in the bloodstream and the others are marginated in close proximity to the endothelium. Most of the marginated pool is in the lung, which has a vascular endothelium surface area. Margination in the post-capillary venules is mediated by selectins which cause a low-affinity neutrophil-endothelial cell interactions that mediates "rolling" of the neutrophils along the endothelium. A variety of signals, including IL-1, TNF-alpha, and other chemokines can cause WBCs to proliferate and leave the BM to enter the circulation. Neutrophil integrins mediate the stickiness of neutrophils to endothelium and are important for chemokine-induced cell activation. Infection causes a marked increase in BM production of neutrophils that marginate and enter tissue. Acute glucocorticoids increase neutrophil count by mobilizing cells from the BM and marginated pool.

During a pre-employment physical and lab eval, 20 YO M is noted to have a prolonged aPTT. On ROS, he denies hx of recurrent mucosal bleeding and has never had an issue with other major bleeding. He has never had any major physical trauma. A famhx is limited because he does not know his biologic famhx. Mixing studies correct the aPTT when normal serum is used. You suspect an inherited hemorrhagic disease such as hemophilia. Which other lab abnormality would you most likely expect to find if this patient has hemophilia? A: low factor VIII activity B: low factor IX activity C: prolonged bleeding time D: prolonged PT E: prolonged thrombin time

A, although B is possible Hemophilia A (absent F VIII) and hemophilia B (absent Factor IX) are indistinguishable clinically. HA accounts for 80% of cases. It has a prevalence in the gen pop of 1:5000 in contrast to HB which has a 1:30,000. The disease phenotype correlates with the amount of residual factor activity and can be classified as severe (<1%), moderate (1-5%), or mild (6-30%). The patient is likely to have a mild form of the disease. Hemophiliacs have a normal bleeding time, PLT count, thrombin time, and PT time. The dx is made by measuring residual factor activity. The prolonged aPTT in hemophilia will be corrected by mixing with normal plasma (that will contain the def factors VIII and IX). Patients with acquired inhibitors will not correct the prolonged aPTT with normal plasma because the defect is antibody mediated

53 YO F seeks evaluation from her PCP regarding primary prevention of CVD and stroke. She has PMH of DM2 for the past 5 years with a known A1c of 7.2%. She does not have HTN or CAD. She has been obese throughout adulthood, and BMI is 33. She is currently perimenopausal with irregular bleeding that last occurred 3 months ago. She is taking Metformin, 1000 mg bid. She has been intolerant of ibuprofen in the past due to GIT upset. She previously smoked 1 ppd from 18-38. She drinks a glass of wine with dinner. Her FamHx is significant for MI in her father at 58 YO, paternal uncle at 67, and paternal grandmother at 62. On maternal side, her mother died of CVA at 62. She is concerned that she should be taking daily ASA as primary prevention of CVD and CVA but is concerned about potential AE. Which is true about ASA therapy? A: ASA is indicated for primary prevention of CVD because she has a strong FamHx and hx of DM B: ASA is only indicated for 2nd prevention of CVD and CVA in women C: because she is not postmenopausal, ASA is not recommended as it will increase menstrual bleeding w/o significantly decreasing the risk of CVD D: her AE to ibuprofen prevents use of ASA because there is high cross reactivity, and she is as risk of bronchospasm E: the risk of major bleeding related to ASA is 1-3% per year, but use of an enteric-coated or buffered ASA will reduce the risk

ASA is the most widely used antiPLT agent worldwide and is cheap and effective for both primary and secondary prevention of CVD. ASA can be recommended for primary in patients who annual estimated risk of CVD is >1%. This includes patients over 40 YO who have 2 or more CVD RF and patients >50 YO who have 1 major RF. The major RF in this patient who is >50 are DM and FamHx. Other RF include obesity and hx of tobacco. ASA is equally effective in men and women, and menopausal status does not impact its efficacy. However, there is a differential effect of ASA in men nd women. In men, it has a greater risk reduction on MI incidence, whereas in women, there is greater risk reduction for CVA. The most common AE is major bleeding at a rate of 1-3% per year. Enteric-coated ASA and buffered preps decrease, but do not eliminate, this risk. The risk of bleeding is higher if administered concurrently with other AC or antiPLT meds. ASA does not cause increased menorrhagia. Finally, ASA should be used with caution in hx of bronchospasm in a/w NSAIDs or ASA. Usually, these patients have hx of asthma and nasal polyposis (Samter's triad). However, this patient reports on GIT upset with ibuprofen. While it is a common AE, it does not denote a true allergy.

64 YO M with CLL and chronic HCV presents for his yearly follow-up. His WBC is stable at 8.3, but his HCT dropped from 35% to 26% and his pLT also dropped from 178 to 69. His initial evaluation should include all of these, except: A: AST, ALT, and PT B: BM biopsy C: Coomb's test D: PBS E: PE

B AIHA and thrombocytopenia are common, and a PBS and a Coomb's test help evaluate their presence. Hypersplenism is also seen in CLL as the spleen sequesters large numbers of circulating blood cells and enlarges. Hence, a careful LUQ exam looking for a palpable splenic tip is the standard of care. This patient is at risk for developing hepatic decompensation as well, given his HCV can also cause anemia and thrombocytopenia. BM infiltration of tumor cells can lead to cytopenias in CLL. However, this is in effect a diagnosis of exclusion. Once these three possibilities are r/o'd, a BM biopsy is a reasonable next step. This initial evaluation before presuming spread of CLL is critical for therapy because each possibility will require different therapy (GCs or retuximab for hemolysis, hepatology referral for liver failure, and splenectomy for symptomatic hypersplenism)

All of the following match the AC with its correct mechanism of action, except: A: Abciximab -- GpIIb/IIIa receptor inhibition B: Clopidogrel -- inhibition of TXA2 release C: Fondaparinux -- inhibition of Factor Xa D: Argatroban -- thrombin inhibition E: Warfarin -- vitamin K dependent carboxylation of coagulation factors

B Anti-platelet and AC agents act by a variety of mechanisms. PLT aggregation is dependent initially on the binding of vWF and PLT glycoprotein IB. This initiates the release of a variety of molecules, including TXA2, and ADP, resulting in PLT aggregation. Glycoprotein IIb/IIIa receptors recognize the AA sequence that is present in adhesive proteins such as fibrinogen. Coagulation occurs by a convergence of different pathways on the prothrombinase complex, which mediates the conversion of fibrinogen to fibrin, thus forming the clot. Factor Xa and Va are two of the essential components of prothrombinase complex. ABCiximab is a monoclonal antibody of human and murine protein that binds to GpIIb/IIIa. It and other inhibitors have been studied extensively in patients with unstable angina, patients with MI, and those undergoing PCI. Clopidogrel acts by inhibiting ADP-induced PLT aggregation. It has been evaluated in many of the same settings either in place of or in conjunction with ASA. Heparin acts to bind factor Xa and activate antithrombin. LMWH primarily act through anti-factor Xa activity. Fondaparinux is a synthetic pentasaccharide that causes selective indirect inhibition of factor Xa. Lepirudin and argatroban are direct thrombin inhibitors. They are indicated in patients with heparin-induced thrombocytopenia. Warfarin acts by inhibiting vitamin K-dependent carboxylation of II, VII, IX, and X.

Fondaparinux may be used to treat all of the patients except: A: 33 YO F weighing 48 kg presenting with PE 2 months after a MVA that resulted in a fractured femur B: 46 YO M with HTn and FSGS with baseline Cr 3.3 presenting with LLE DVT. He weighs 82 kg. C: 57 YO F had an aortic valve replacement 7 days ago. The PLT pre-op 320. On day 7, the PLT 122 D: 60 YO M presents to hospital with CP and ST depression in II, III, and AVF. Troponin I is 2.32 E: 68 YO M has undergone uncomplicated R total hip replacement

B Fondaparinux is a direct factor Xa inhibitor that is synthetic analogue of the pentasaccharide sequence found in Heparin. A smaller compound the either UFH or LMWH, fondaparinux acts by binding antithrombin and catalyzing factor Xa inhibition. At only 5 polysaccharide units, it is too small to bridge antithrombin to thrombin and does not potentiate thrombin inhibition. It is given SubQ and has 100% bioavailability without plasma protein binding. Like LMWH, it has a predictable AC effect and monitoring factor Xa levels is not required. It is excreted unchanged in the urine. It is absolutely contra in those with a Cr CL <30 and should be used with caution in individuals with Cr CL <50. The individual in B has a CL of 32 mL/min and should not receive fondaparinux.

Which of the following regarding malignant spinal cord compression (MSCC) is true? A: less than 50% of patients who are treated while ambulatory will remain ambulatory B: neurologic abnormalities on PE are sufficient to initiate high dose GC C: neuro findings are often present before pain D: renal cell carcinoma is the MC cause E: the lumbosacral spine is the MC affected site

B MSCC syndrome is defined as compression of the spinal cord and/or caudal equina by an extradural tumor mass. The minimum radiologic evidence for cord compression is compression of the theca at the level of clinical features. However, radiology is not necessary in a patent whose PE suggests cord compression. These patients should receive immediate high dose dexamethasone (24 mg IV q/6). Cancers that most often cause MSCC include prostate, lung, and breast. RCC, lymphomas, and melanomas may also cause it. The MC affected site is the thoracic spine (70%), followed by sacral (20%). Pain is usually present for days or months before the neurologic ssx. Some 75% who are ambulatory at the time of diagnosis will remain ambulatory, whereas <10% of those who present with paraplegia will regain the ability to walk despite treatment

patients from which of the following regions need not be screen for G6PD deficiency when starting a drug which carries a risk for G6PD mediated hemolysis? A: Brazil B: Russia C: SE Asia D: Southern Europe E: Sub-Saharan Africa F: None of these

B RBCs utilize glutathione produced by the hexose monophosphate shunt (HMP) to compensate for increased production of ROS (oxidative stress), usually induced by drugs or toxins. Defects in G6PD are the MC congenital hexose monophosphate shunt defect. If the RBC is unable to maintain an adequate level of glutathione during oxidant stress, Hgb ppt in the RBC, producing Heinz bodies. Because the G6PD gene is on the X chromosome, almost all afflicted patients are males. Deficiency is widely distributed throughout the regions that are currently or were once highly malarial endemic. It is common in males of African, African-American, Sardinian, and Sephardic descent. In most people with the deficiency, there is no evidence of symptomatic disease. However, infection, ingestion of fava beans, or exposure to an oxidative agent (drug or toxin) can trigger an acute hemolytic event. Bite cells, Heinz bodies, and bizarre poikilocytes may be evident on smear. The drugs which most commonly ppt crisis include Dapsone, SMX, primaquine, and nitrofurantoin. The anemia is often severe with rapid onset after drug ingestion, and renal failure can occur.

All of these are characteristic of tumor lysis syndrome, except: A: hyperkalemia B: hypercalcemia C: lactic acidosis D: hyperphosphatemia E: hyperuricemia

B TLS is a well recognized clinical entity that is characterized by metabolic derangements 2/2 destruction of tumor cells. Lysis of cells causes the release of intracellular pools of phosphate, potassium, and nucleic acids, leading to hyperphosphatemia and hyperuricemia. Lactic acidosis results. The increased urine acidity may promote the formation of uric acid nephropathy and subsequent renal failure. Hyperphosphatemia promotes a reciprocal depression in the serum calcium. This hypocalcemia may result in severe neuromuscular irritability and tetany.

A Sudanese refugee is brought to clinic for abdominal pain. He has had intermittent fevers for months and has lost considerable weight. He was previously a guard for a refugee camp in Sudan and worked the night shift exclusively. On exam, he is severely malnourished with temporal wasting. He has massive splenomegaly but no palpable LAD. Oropharynx shows no thrush. Labs reveal an anemia, neutropenia, and thrombocytopenia. Skin exam shows no discrete lesions but you and the patient notice that the skin appears gray throughout. Malaria smears are negative and HIV is negative. CXR is wnl. What is the most likely diagnosis? A: cirrhosis B: Kala-azar (visceral leishmaniasis) C: Kaposi sarcoma D: Miliary TB E: SCD

B The ddx for a patient presenting with visceral leishmaniasis is broad and includes diseases that cause fever or organomegaly. Characteristic findings include hx of exposure to sandflies at night or darkening of the skin on PE. The skin discoloration is usually seen in end stage cachetic patients. Miliary TB is on the ddx but would be unlikely with a normal CXR. Cirrhosis of the liver may present this way although persistent fevers would be uncharacteristic. The visceral form of KS may present and can be seen in HIV negative patients who is otherwise malnourished or immunosuppressed. KS would be less likely than visceral leishmaniasis given the exposure hx and characteristic end-stage finding of skin discoloration. SCD causes autosplenectomy, but not splenomegaly.

You are consulted for a 31 YO M with prolonged bleeding after an oral surgery procedure. He has no prior hx of bleeding diathesis or FamHx of bleeding disorders. The patients PMH is remarkable for infection with HIV, with CD4 51. The exam is remarkable only for spotty LAD. The PLT 230. His INR is 1.5. aPTT is 40s. PBS shows no schistocytes and is otherwise wnl. A 1:1 mixing study corrects both conditions immediately after 2h incubation. Fibrinogen is wnl. Thrombin time is prolonged. What is the dx? A: DIC B: dysfibrinogenemia C: Factor V def D: liver disease E: Factor XIII def

B fibrinogen is a 340 kdA dimeric molecule made up of 2 sets of (3) covalently linked polypeptide chains. Thrombin cleaves multiple peptides to produce fibrin monomers that factor XIII stabilizes by cross-linking. Although fibrinogen is needed for PLT aggregation and fibrin formation, even severe fibrinogen def such as afibrinogenemia produces mild, rare bleeding episodes, most often s/p surgery. Dysfibrinogenemia refers to a constellation of disorders that involves mutations that alter the release of fibrinopeptides, affect the rate of polymerization of fibrin monomers, or alter the sites of the fibrin cross linking. Dysfibrinogenemias are either inherited in AD fashion or acquired. Patients with liver disease, hepatomas, AIDS, and lymphoproliferative disorders may develop an acquired form. The presence of altered PTT and PT/INR reflects an abnormality in the coagulation from the prothrombinase complex downstream to fibrin. Correction with a mixing study eliminates factor inhibition as the cause of the disorder. Other causes of prolonged PT and PTT include factor V or X, afibrinogenemia or dysfibrinogenemia, and consumption of coagulation factors from DIC. The absence of schistocytes from the smear make DIC unlikely. The thrombin time tests the interaction with thrombin directly on fibrinogen. Its prolongation indicates an abnormality with that interaction and suggests a diagnosis of dysfibrinogenemia. Factor XIII def is a bleeding disorder that manifests in childhood and is not c/w this presentation.

56 YO patient inquires about screening for CRC. He has no RF for CRC, other than age. Which of these statements is true about which screening test you recommend? A: 50% of patients with positive FOBT have CRC B: one-time colonoscopy detects more advanced lesions that one-time FOBT with sigmoidoscopy C: perforation rates for sigmoidoscopy and colonoscopy are equivalent D: sigmoidoscopy has not been shown to reduce MR E: virtual colonoscopy is as effective as endoscopic colonoscopy for detecting polyps <5 mm in size

B for CRC screening, the (3) major preventive societies (ACS, USPSTF, and Canadian Task force) recommend sigmoidoscopy, colonoscopy, or FOBT starting at 50 YO. DRE is not recommended. FOBT has a high FP rate; 2-10% of those with positive result have CRC, and ~25% have adenomas. Sigmoidoscopy has been shown to reduce MR, and the recommended interval is 5 years. Sigmoidoscopy carries a perforation risk of 1/1000, while the risk of colonoscopy is 3x greater. Colonoscopy detects more advanced lesions and is the screening test of choice in subjects who are high risk. Virtual colonoscopy using CT imaging can detect adenomatous polyps, compares favorably with endoscopy colonoscopy for polyps >8mm in size, and may be effective in average risk adults. It is not as sensitive as endoscopy for small <5mm polyps

A primary tumor of which of these organs is the least likely to mets to bone? A: breast B: colon C: kidney D: lung E: prostate

B mets to bone are MC that primary bone tumors. Prostate, breast, and lung account for 80% of all bony mets. Tumors from kidney, bladder, and thyroid and lymphomas and sarcomas also commonly mets to bone. Mets usually spread hematogenously. In decreasing order, the MC sites of bone mets include vertebrae, proximal femur, pelvis, ribs, sternum, proximal humerus, and skull. Pain is the MC ssx. Hypercalcemia may occur. Lesions may be osteolytic, osteoblastic, or both. Osteoblastic lesions are MC a/w a higher level of +ALP.

a healthy 62 YO F returns to your clinic after undergoing routine colonoscopy. Findings include (2) 1.3 cm sessile (flat based), villous adenomas in her ascending colon that were removed during the procedure. What is the next step in her management? A: colonoscopy in 3 months B: colonoscopy in 3 years C: colonoscopy in 10 years D: CT of the abdomen E: partial colectomy F: reassurance

B most CRC arise from adenomatous polyps. Only adenomas are premalignant, and only a minority of these become malignant. Most polyps are asymptomatic, causing occult bleeding in <5% of patients. Sessile (flat based) polyps are more likely to become malignant than pedunculated (stalked) polyps. Histologically, villous adenomas are more likely to become malignant than tubular adenomas. The risk of containing invasive carcinoma in the polyp increases with size with <2% in polyps <1.5 cm, 2-10% in polyps 1.5-2.5 cm, and 10% in polyps >2.5 cm. This patient had two polyps that were high risk histology (villous) and appearance (sessile), but only moderate risk by size (<1.5 cm). Polyps, particularly those >2.5 cm in size, sometimes contain cancer cells but usually progress to cancer quite slowly over ~ 5 year period. Patients with adenomatous polyps should have follow-up colonoscopy or radiographic study in 3 years. If no polyps are found on initial study, the test should be repeated in 10 years. CT is only warranted for staging if there is diagnosis of CRC, not for polyps alone.

65 YO M IN ED BIBEMS after his daughter found him to be incoherent earlier today. She last spoke with him yesterday, and at that time, he was c/o 2 days of myalgia, HA, and fever. He had attributed it to an URTI and did not seek evaluation from PCP. Today, he did not answer when she called his home, and she found him in bed smelling of urine. He was minimally arousable but appeared to be moving all extremities. PMH is significant for HTN, HLD, and COPD. He was evaluated 2 weeks previously for a TIA and after an episode where he had numbness and weakness of his LE and LLE which resolved over 6 hours w/o intervention. His currents meds are ASA 81 mg qd, clopidogrel 75 mg qd, Atenolol 100 mg q/d, Atorvastatin 20 mg q/d and Tiotropium q/d. He is allergic to Lisinopril, which caused angioedema. He is a former smoker and drinks ETOH rarely. On PE, he is obtunded and minimally arousable. He is febrile at 38.9C. His BP is 159/96, and HR 98, RR 24 with 95% on RA. He has minimal scleral icterus. The oropharynx reveals dry mucous membranes. His CVS, Chest, and ABD are wnl. There are no rashes. His neuro exam is difficult to obtain. There are no CN findings. He resists movement of his extremities but has normal strength. DTR are brisk, 3+ and equal. Labs show Hgb 9.3, HCT 29, WBC 14, PLT 42. DDX shows 83% PMNs, 2% bands, 6% lymphocytes, and 9% monocytes. Na 145, K 3.8, Cl 113, HCO3 19, BUN 68, and Cr 3.4. Bilirubin 2.4, and LDH 450. PBS shows diminished PLTs and many schistocytes. What is the next step? A: d/c clopidogrel B: d/c clopidogrel and start plasmapheresis C: initiate therapy with IV-immunoglobuin D: obtain head CT and start treatment with factor VIIa, if subarachnoid hemorrhage is seen E: perform LP and start broad-spectrum ABX with Ceftazidime and Vancomycin

B patient has evidence of TTP from clopidogrel manifested as: 1. AMS 2. fever 3. AKI 4. thrombocytopenia 5. microangiopathic hemolytic anemia the PBS shows anisocytosis with schistocytes and PLT clumping c/w this disease. Clopidogrel is a thienopyridine anti-PLT agent that is known to be a/w life-threatening hematologic effects, including neutropenia, TTP, and aplastic anemia. The true incidence of TTP a/w thienopyridine use is unknown, but it occurs with both clopidogrel and ticlopidine use. When compared to ticlopidine, TTP a/w clopidogrel use occurs earlier (often within 2 weeks) and tends to be less responsive to therapy with plasmapheresis. In addition, individuals with TTP a/w Clopidogrel generally have a higher PLT count and Cr and their TTP is less likely a/w ADAMTS13 deficiency, a vWF-cleaving protease implicated in the pathogenesis of idiopathic TTP. The mortality of TTP a/w thienopyridines is approximately 25-30%.

42 YO M presents with RUQ pain. He was found to have multiple masses in the liver which were malignant on H&E. Your initial hx, physical, and labs (including PSA) are unrevealing. Lung, ABD, and pelvic CT scans are wnl. He is an otherwise healthy individual with no chronic medical problems. Which IHC markers should be obtained? A: AFP B: cytokeratin C: leukocyte common antigen D: thyroglobulin E: thyroid transcription factor 1

B patients with cancer from an unknown primary site present a common diagnostic dilemma. Initial evaluation should include H, PE, imaging, and blood studies based on gender (e.g., PSA in men, mammogram in women). IHC using antibodies to specific cell components may help elucidate the site of the primary tumor. additional test may be helpful based on appearance under microscopy and/or the results of the cytokeratin stains. In cases of unknown primary, cytokeratin staining is usually the first branch point from which the tumor lineage is determined. Cytokeratin is positive for carcinoma, since all epithelial tumors contain this protein. Subsets of cytokeratin, such as CK7 and CK20, may be useful to determine the etiology of the primary tumor. Leukocyte common antigen, thyroglobulin, and thyroid transcription factor 1 are characteristics of lymphoma, thyroid cancer, and lung or thyroid cancer, respectively. AFP is used in germ cell, stomach, and liver cancer.

A woman wants advice on Pap smears. She is 36 YO and is monogamous with her husband since married 3 years ago. She has had normal Pap smears q/year for the past 6 years. She would like to avoid the yearly test. What is your advice to this patient, based on current screening guidelines? A: she may d/c screening at age 50 if she has normal pap smears for the previous 10 years B: she may extend the screening interval to 1x/2-3 years C: she may extend the screening interval to 1x/5 years if she agrees to use barrier protection D: she may d/c Pap smears if she receives the HPV vaccine E: the only indication to cease Pap smears is if she would have a total hysterectomy

B the ACS recommends yearly Pap smears starting at age 21, regardless of onset of intercourse. The USPSTF recommends testing q/1-3 years for women 18-65 YO. At age 30, women who have had 3 successive years of normal test results may extend their interval screening to 2-3 years. An upper age limit at which screening ceases to be effective is unknown, however, women >70 YO may choose to stop testing if they have had normal Pap smears for 10 years. Women who have no cervical remnant (i.e, total hysterectomy) do not require Pap smears. Current recommendations advise continued Pap smears even after receiving HPV vaccination given that the vaccine does not protect against all strains of HPV which cause cervical cancer. The vaccine protects against about 70% of cervical cancer. `

A patient comes for follow-up for evaluation for elevation in HCT. You suspect PV based on hx of aquagenic pruritus and splenomegaly. What set of labs are c/w PV? A: elevated RBC mass, high serum EPO, normal O2 sats B: elevated RBC mass, low serum EPO, normal O2 sat C: normal RBC mass, high serum EPO, low arterial O2 sat D: normal RBC mass, low serum EPO, low arterial O2 sat

B the first step in dx of PV is to document elevated RBC mass. A normal RBC mass suggests spurious polycythemia. Next, serum EPO levels should be measured. If EPO are low, the dx is PV. Confirmatory tests with Jak-2 mutation analysis, leukocytosis, and thrombocytosis. Elevated EPO are seen in normal physiologic response to hypoxia as well as in autonomous production of EPO. Further steps in the workup include evaluation for hypoxia with ABG, consideration of smoker's polycythemia (elevated carboxyHgb) and disorders of Hgb affinity for oxygen. Low EPO with low O2 sats suggest inadequate renal production (renal failure). High RBC mass and high EPO levels with normal O2 sat may be seen in autonomous EPO production, such as in RCC.

70 YO M is admitted to CCU for c/o chest pressure occurring at rest radiating to his L arm with associated diaphoresis and pre-syncope. His admission EKG showed ST depressions in V4-V6. The CP and EKG changes resolve with sublingual nitroglycerin. He is treated with IV-Heparin, ASA, Metoprolol, and Lisinopril. His cath shows 90% occlusion in the LAD, 80% occlusion of the distal circumflex, and 99% occlusion of the RCA. He remains in CCU awaiting CABG. He has a hx of rheumatic heart disease and underwent mitral valve replacement at 58 YO. On admission, Hgb 12.2, HCT 37, WBC 9.8, PLT 240K. His Cr is 1.7. On Day 4, his Hgb 10, HCT 31, WBC 7.6, and PLT 112K. His Cr has risen to 2.9 after the catheterization. What is the most appropriate treatment at this time? A: c/w Heparin and give PLT transfusion B: d/c Heparin and start Argatroban C: d/c Heparin and start Lepirudin D: d/c Heparin and start Warfarin E: send serum to assess for the presence of PF4 IgG antibodies and c/w Heparin

B the most likely dx in this patient is HIT, and heparin should be stopped immediately while c/w A.C. with the direct thrombin inhibitor, Argatroban. HIT should be suspected in individuals with a fall in PLT count >50%, of pretreatment levels. Usually the fall in pLT count occurs 5-13 days after starting heparin, but it can occur earlier if there is prior exposure to heparin (this man had valve replacement). While a PLT count of <100K is highly suggestive e of HIT, in most individuals, the PLT count rarely falls this low. HIT is caused yb IgG antibodies directed against antigens on PF4 (platelet factor 4) that are exposed when heparin binds to this protein. IgG antibody binds simultaneously to heparin-PF4 complex and the Fc receptor on PLT surface and causes PLT activation, resulting in a hypercoagulable state. Individuals with HIT are at increased risk of both arterial and venous thromboses, although DVT are much more common. Demonstration of abs directed against the complex is suggestive of, but not sufficient for, dx because these abs can be present in the absence of clinical HIT. The serotonin release assay is the most specific test for determining if HIT is present. This assay determines the amount of serotonin released when washed PLTs are exposed to patient serum and varying concentrations of heparin. In HIT, addition of the serum to the test causes PLT activation and 5-HT3 release due to the presence of heparin-PF4 abs. However, treatment should not be delayed until definitive dx as there is risk of thrombotic events if heparin is c/w. The risk is increased for about 1 month s/p heparin. Thus, all patients with HIT should be continued on A.C. until the thrombosis risk decreases. Patients should not be switched too LMWH. While the incidence of HIT is lower with LMWH, there is crossreactivity with complex antibodies, and thrombosis can occur. Choic of AC should be direct thrombin inhibitor or a factor Xa inhibitor. The direct thrombin inhibitors include lepirudin, argatroban, and bivalirudin. In this patient, Argatroban is the best choice because the patient has developed AKI in a/w contrast dye for his cath. Argatroban is hepatic ally metabolized and is safe to give in AKI, whereas Lepirudin is renal metabolized. Dosage of lepirudin in AKI is unpredictable, and lepirudin should not be used in this setting. The factor Xa inhibitors, fondaparinux or danaparoid, are also possible treatments, but due to renal metabolism, are also CONTRA. Finally, warfarin is contra for the sole treatment for HIT as the fall in vitamin K dependent factors, especially Protein c, can further increase the risk of thrombosis and trigger skin necrosis.

Which of the following is sufficient to make a definitive dx of porphyria? A: appropriate clinical scenario including positive fam hx B: evidence of an enzyme deficiency or gene defect C: labs in blood indicating accumulation of porphyrin precursors D: labs in urine indicating accumulation of porphyrin precursors at the time of ssx E: labs in stool indicating accumulation of porphyrin precursors at the time of ssx

B the porphyrias are a group of metabolic disorders resulting from a specific enzyme deficiency in the heme synthesis pathway. All are inherited except porphyria cutanea tarda (PCT), which is usually sporadic. They are classified as either erythropoietic or hepatic, depending on the site of overproduction or accumulation of porphyrins or precursors. The predominant ssx of the hepatic porphyrias (e.g., AIP, PCT) are neurologic and include pain, neuropathy, and mental disturbances. The erythropoietic porphyrias usually precent with cutaneous photosensitivity at birth. However, PCT which is hepatic, usually presents with skin lesions. The genetic mutations which cause each type of porphyria have been elucidated, and demonstration of a specific gene defect or resulting enzyme deficiency is required for definitive dx. Clinical ssx of porphyrias are notoriously nonspecific with great overlap. Labs of fecal, urinary, or plasma protoporphyrins, porphobilinogens, or porphyrins during a crisis will help guide diagnosis but require further testing for confirmation. The ssx of many of the porphyrias are exacerbated by a wide variety of drugs.

All of these are obstacles to the more widespread application of stem cells for regenerative medicine, except: A: controlling the migration of transplanted SCs B: ID'ing diseases suitable for SC based therapy C: ID'ing the pathways for ddx SC into specific cell types D: overcome ethical concerns over their harvest and use E: predict the response of cells to the environment of the diseased organ

B there are many attractive targets for which to use SC as regenerative therapies (e.g., MI, DM1, Parkinson's). All other options represent a few of the obstacles that are yet to be overcome. SC therapies raise important questions about the definition of human life and have raised issues of justice and safety regarding care of patients. It is clear that the resolution of these ethical issues will require multidisciplinary discussion between scientist, physicians, lawmakers, and general public

16 YO M has recurrent thigh hematomas. He has been active in sports all his life and has (3) episodes of limb-threatening bleeding with compartment syndrome. A FamHx is notable for maternal grandfather with a similar bleeding history. Paternal fam hx is not available. Labs reveal normal PLT count, normal aPTT (22s) and prolonged PT (25s). He takes no meds. What is the most likely reason for his coagulation disorder? A: Factor VIII def B: Factor VII def C: Factor IX def D: Prothrombin def E: Surreptitious warfarin ingestion

B this patient has a coagulation disorder characterized by recurrent bleeding episodes into closed spaces with an inheritance pattern suggestive of recessive or XL pattern. An isolated prolonged PT suggests Factor VII deficiency, which is inherited in AR pattern. The thrombin time will also be normal in these cases. While Hemophilia A (factor VIII def) and hemophilia B (factor IX) are the most common inherited factor deficiencies, these disorders do not cause an isolated prolonged PT. They will cause a prolonged aPTT with a normal PT. Both are inherited in XL pattern. PT def is a rare AR disorder that will cause prolonged aPTT, PT, and thrombin. Ingestion of Warfarin may also cause this scenario but is less likely given the inheritance pattern.

36 YO African-American woman with SLE presents with acute onset of lethargy and jaundice. On initial exam, she is tachycardic, hypotensive, appears pale, is SOB, and is somewhat difficult to arouse. PE reveals splenomegaly. Her initial Hgb is 6, WBC is 6.3 and PLT 294. Her total bilirubin is 4, reticulocyte count is 18%, and haptoglobin is not detectable. Renal function is wnl, as is UA. What would you expect on PBS? A: macrocytosis and PMNs with hypersegmented nuclei B: microspherocytes C: schistocytes D: sickle cells E: target cells

B this patient's lupus and her rapid development of truly life-threatening hemolytic anemia are both very suggestive of AIHA. Dx is made by a positive Coomb's test documenting antibodies to the RBC membrane, but smear will often show microspherocytes, indicative of the damage incurred to the RBCs in the spleen. Schistocytes are typical for microangiopathic hemolytic anemias such as HUS or TTP. The lack of thrombocytopenia makes these diagnoses considerably less plausible. Macrocytosis and PMNs with hypersegmented nuclei are very suggestive of B12 deficiency, which causes a more chronic, non-life-threatening anemia. Target cells are seen in liver disease and thalassemia. Sickle cell anemia is a/w aplastic crises, but she has no known diagnosis of sickle cell disease and is showing evidence of EPO response based on the presence of elevated reticulocyte count

Which of these statements describes the relationship between testicular tumors and serum markers? A: pure seminomas produce AFP or b-HCG in more than 90% of cases B: more than 40% of nonseminomatous germ cell tumors produce no cell markers C: both b-hcg and AFP should be measured in following the progress of a tumor D: measurement of tumor markers the day after surgery for localized disease is useful in determining completeness of the resection E: b-hcg is limited in its usefulness as a marker because it is identical to human luteinizing hormone

C 90% of persons with non-seminomatous germ ell tumors produce either AFP or b-hcg; in contrast, persons with pure seminomas usually produce neither. These tumor markers are present for some time after surgery; if the pre-surgical levels are high, 30 days or more may be required before meaningful post-surgical levels may be obtained. The half lives of AFP and b-hcg are 6 days and 1 day, respectively. After treatment, unequal reduction of b-hcg and AFP may occur, suggesting that the two markers are synthesized by heterogenous clones of cells within the tumor; thus, both markers should be followed. B-hcg is similar to LH except for its distinctive beta subunit.

Chemoprevention strategies for cancer have met with varying levels of success. Which of these pairings correctly identifies an effective chemoprevention strategy with its target effect? A: ASA: colon cancer B: B-carotene: lung cancer C: Calcium: adenomatous GI polyps D: isotretinoin: oral leukoplakia E: tamoxifen: endometrial cancer

C Chemoppx involves the use of a specific natural or synthetic chemical agent to reverse, suppress, or prevent carcinogenesis before the development of invasive malignancy. Calcium, by binding to luminal FFAs and bile, may reduce GI endothelium proliferation. Ca supplementation decreases the risk of adenomatous polyps by up to 20%. Trials with cancer-incidence endpoints are currently underway. High doses of relatively toxic istretinoin caused regression of premalignant oral leukoplakia lesions; however, lower doses were not effective in preventing head and neck cancer. It also did not ppx 2nd malignancies in patients cured of early stage NSCLC. B-carotene has been investigated for the prevention of lung cancer in two trials. Both showed harm from B-carotene. ASA had no effect on CRC incidence in a 6-year trials. COX 2 inhibitors have been shown to reduce recurrence rates for polyps in FAP. The effects on CRC in sporadic cases were initiated, but were complicated by the association of these drugs with increased CVD. Tamoxifen is used for primary ppx of breast Ca among those at high risk. It is a/w a small increase in the risk of endometrial cancer.

You consult on a 34 YO M with thrombocytopenia. He sustained a MVA 10 days ago, resulting in shock, internal bleeding, and ARF. An ex lap was performed which showed a ruptured spleen requiring a splenectomy. He also underwent an ORIF of L femur. PLT 260 on admission. Today it is 68. His meds are oxacillin, morphine, and SQ heparin. On exam, the VS are wnl. The exam is significant for an ABD scar which is clean and healing. The patient's L LE is in a large cast and is elevated. The R LE is swollen from the calf downward. US of the R LE shows a DVT. Antiheparin abs are positive. Cr 3.2. What is the most appropriate step? A: d/c Heparin B: Stop Heparin and start Enoxaparin C: Stop Heparin and start Argatroban D: Stop Heparin and start Lepirudin E: Observe the patient

C Heparin-induced thrombocytopenia (HIT) is common in patients who receive heparin products. Because the risk of death is significantly increased in patients with HIT type II and thrombosis if no AC is given, observation or simply d/c Heparin is not an option. Although enoxaparin and other LMWH have less of a propensity to cause HIT, they are cross reactive in patients who already have HIT and thus are CONTRA. Direct thrombin inhibitors are DOC. Lepirudin is a recombinant direct thrombin inhibitor. It may be given IV or SQ. It is excreted through the kidney and lacks antidote. Therefore, it is contraindicated in renal insufficiency. Argatroban is another direct thrombin inhibitor. Because it is hepatic ally metabolized, it is a reasonable option in patients with HIT and renal insufficiency.

22 YO F pregnant of North European descent presents 3 months into her first pregnancy with extreme fatigue, pallor, and icterus. She reports being previously healthy. On exam, Hgb 8, reticulocyte 9%, indirect bilirubin 4.9, and serum haptoglobin is not detectable. Her PE is notable for splenomegaly and a normal 3 month uterus. What is the diagnosis? A: colonic polyp B: G6PD deficiency C: hereditary spherocytosis D: parvoB19 infection E: TTP

C Hereditary spherocytosis is a heterogeneous RBC membrananopathy that can be congenital (AD) or acquired; it is characterized by predominantly extravascular hemolysis in the spleen due to defects in the membrane structural proteins. This spleen-mediated hemolysis leads to the conversion of classic biconcave RBCs on smear to spherocytes. Splenomegaly is common. This disorder can be severe, depending on the site of mutation, but is often overlooked until some stressor such as pregnancy leads to multifactorial anemia, or an infection such as PB19 transiently eliminates RBC production all together. The PBS shows microspherocytes, small densely staining RBCs that have lost their central pallor. Acute treatment is with transfusion. G6PD deficiency is a cause of hemolysis that is usually triggered by the presence of an offending oxidative agent. The PBS may show Heinz-Bodies. PB19 infection may cause pure red cell aplasia. The presence of active reticulocytes and labs c/w hemolysis are not compatible with that diagnosis. Chronic GIT blood, such as from a colonic polyp, would cause a microcytic, hypo chromic anemia w/o evidence of hemolysis (indirect bilirubin, haptoglobin abnormalities)

56 YO F diagnosed with CML, Ph+. Her presenting WBC 127, and her ddx shows <2% blasts. Her HCT is 21 at diagnosis. She is asymptomatic except for fatigue. She has no siblings. What is the best initial therapy? A: allogenic BMT B: autologous SCT C: imatinib mesylate D: IFN-alpha E: leukapheresis

C Imatinib is a TK inhibitor that acts to decrease the activity of the BCR-ABL fusion protein which results form the reciprocal translocation of the 9:22. It acts as a competitive inhibitor of the ABL kinase at its ATP binding site and thus leads to inhibition of tyrosine phosphorylation of proteins in the BCR-ABL signal transduction. It results in heme remission in 97% of treated individuals at 18 months and cytogenetic remission of 76%. This is compared to traditional CTX of IFNa and cytarabine, which resulted in 69% heme remission and 14% cytogenetic. More than 87% of individuals who achieved cytogenetic remission had not developed progressive disease at 5 years. This drug taken orally has limited AE which include nauseas, fluid retention, diarrhea, and skin rash and is usually well tolerated. If individuals do not achieve heme remission by 3 months or complete cytogenetic remission by 12 months, it is recommended that they proceed to allogenic BMT. While imantinib is the best initial therapy, individuals who have a well-matched BM donor may proceed to early allogenic transplant, particularly if the individual is <18 YO. This is done because younger individuals generally have better outcomes following BMT than older individuals, and the durability of response to imatinib is not known at this time. IFNa was previously given as first line CTX if the BMT was not an option, but it has been replaced by imatinib. Autologous SCT is not currently used for CML as there is no reliable way to select residual normal hematopoietic progenitor cells. Clinical trials using autologous SCT are currently underway to determine if this treatment may be possible following control of disease with imatinib. Leukopheresis is used for control of WBC counts when the patient is experiencing cx such as respiratory failure or cerebral ischemia related to high WBC count.

23 YO F diagnosed with LE DVT. Which of the following medical conditions represents a CONTRA to therapy with LMWH? A: pregnancy B: obesity C: dialysis-dependent renal failure D: uncontrolled DM E: jaundice

C LMWH are cleared renally, and these drugs have been described as causing significant bleeding in patients on HD. They should not be used in patients with HD-dependent renal failure. They are class B drugs for pregnancy and dosage is weight based. Their utility is not affected by DM or hepatic dysfunction. Thrombocytopenia is a rare AE of both unfractionated heparin and LMWH, but LMWH should not be used in someone with a documented hx of heparin-induced thrombocytopenia.

65 YO M seeks evaluation for nasal congestion, HA, and dysphagia, most often when he lies supine for sleeping. These ssx have been slowly worsening for the past month. He has no nasal d/c or fever. On ROS, he reports recent hoarseness or dizziness. His PMH is significant only for mild HTN. He worked as a roofing contractor and smoked 1 ppd since 16 YO. On PE, you note facial edema. His oropharynx is also mildly edematous, and the tonsils are wnl. His external and internal jugular veins are engorged b/l, and there are prominent veins on the anterior chest. Chest percussion reveals dullness in the R base with decreased tactile fremitus. A CXR shows a RUL mass that on biopsy is c/w NSCLC. all of the following treatments may help him, except: A: CTX B: diuretics C: glucocorticoids D: XRT E: venous stenting

C SVC syndrome is the clinical manifestation of SVC obstruction with severe reduction in VR from the head, neck, and UE. Small cell and squamous cell account for 85% of all cases of malignant SVC obstruction. Common c/o neck and facial swelling with SOB. Other ssx include hoarseness, tongue swelling, HA, nasal congestion, epistaxis, hemoptysis, dysphagia, pain, dizziness, syncope, and lethargy. Temporizing measures include diuretics, low-salt diet, oxygen, and head elevation. GC may be effective for shrinking the size of lymphomatous masses, but they are no benefit for primary lung cancer. XRT is the primary treatment of SVC 2/2 NSCLC. CTX is most effective for small cell, lymphoma, or germ cell tumors. Some NSCLC are responsive to new CTX agents. Intravascular stunting is effective for palliation and may be considered to prevent recurrence.

A new screening test for thyroid cancer has been introduced into the population. In the first year, 1000 positive tests lead to correct ID of thyroid cancers in the screen group. Over the next year, 250 cases of cancer are detected among those who initially had a negative test. What is the sensitivity of the new test? A: 25% B: 67% C: 80% D: Not enough info

C The sensitivity of the test is a numerical description of the test's ability to detect the disease when it is present. It is the proportion of people with the condition who also test positive. In this example, 1000 people test positive using the test. The number of persons who actually have the condition is 1250, yielding 80%

54 YO F with Afib is A.C. with warfarin, 5 mg daily. She developed a UTI that her PCP has treated with Ciprofloxacin, 250 mg po bid for 7 days. She presents to the ED today c/o blood in her urine and easy bruising. Her PE shows ecchymoses on her arms. Her urine is bloody in appearance, but no clots are present. After flushing the bladder with 100 mL of sterile saline, the urine returns with a slight pink hue only. UA shows 3-5 WBCs and many RBCs. There are no bacteria present. The INR is 7.0. What is the best approach? A: administer vitamin K 10 mg IV B: administer vitamin K 2 mg SQ C: administer vitamin K 1 mg sublingual D: hold warfarin until the INR falls to 2.0 E: transfuse 4 U of FFP

C Warfarin is the most widely use oral A.C. Its mechanism of action is to interfere with production of Vitamin-K dependent pro-coagulant factors (prothrombin, factor VII, IX, and X) and anticoagulant factors (Protein C and S). Warfarin accumulates int eh liver when it undergoes oxidative metabolism by CYP2C9 system. Multiple medications can interfere with the metabolism of warfarin by this system causing both over- and underdosing of Warfarin. This patient has recently been treated with a fluoroquinolone antibiotic which is known to increase the PT and INR if the warfarin dose is not adjusted. When the INR >6, there is a greater risk of bleeding cx. However, if no evidence of bleeding is present at presentation, it is safe to hold warfarin and allow the INR to fall gradually into the therapeutic range before reinstitution of therapy. In this patient, however, there is evidence of minor bleeding cx warranting treatment. She likely has developed a degree of hemorrhagic cystitis due to over A.C. in the setting of a UTI, which had already inflamed the bladder lining. In addition, she had developed multiple ecchymoses. Thus, treatment of the elevated INR is indicated. When the INR falls between 4.9 - 9.0, a po dose of Vitamin K, 1 mg, is usually adequate to correct the INR w/o conferring resistance, as evidenced by decreased sensitivity to oral warfarin for an extended period. When a more rapid correction of A.C. is needed, Vitamin K can be given IV or IM. However, there is risk of major anaphylaxis, shock, and death. This can be minimized by delivering the drug slowly at a rate of <=1mg/min. Additionally, FFP is indicated to replete coagulation factors when there is significant bleeding in the setting of an elevated INR. While the SQ route for delivery has long been a primary route of correction, a meta-analysis has shown that SQ is no better than placebo and inferior to the oral and IV routes, which have similar efficacy.

a woman with advanced breast cancer being treated with tamoxifen presents to the ED with nausea/vomiting. She has been tolerating her treatment well but in the last 3 days noticed nausea/vomiting and ABD pain. Her SSX are not related to food intake, and she is having normal bowel movements. She has no fevers or rashes. Her meds include tamoxifen, alendronate, megestrol acetate, and a MVA. ABD exam reveals very mild tenderness diffusely, and there is no rebound tenderness. BS are wnl. Plain XR and CT of the abdomen are unremarkable. Labs show a normal WBC. Na is 130, K is 4.9, Cl 99, HCO3 29, BUN 15, Cr 0.7. What is the next most appropriate step in this patient's management. A: antiemetic prn B: laparoscopy C: serum cortisol D: small bowel follow through E: upper endoscopy

C abdominal pain can be a sign of an oncologic emergency, both obstructive or metabolic. The ddx is broad; however, when there is obstruction, constipation and colicky ABD pain are prominent. The pain may also be exacerbated postprandially. Normal imaging, moreover, suggests the abnormality is metabolic or may be due to peritoneal mets too small to be seen on standard imaging. Adrenal insufficiency is suggested by mild hyponatremia and hyperkalemia, the hx of breast cancer and use of megestrol acetate. AI may go unrecognized because the symptoms of n/v, orthostasis, or hypotension may be mistakenly attributed to progressive cancer and/or therapy.

52 YO M admitted with recurrent hemarthroses of his knees. He is an electrician who is still working but over the last year has had recurrent hemarthroses requiring surgical evaluation. Before 1 year ago, he has no medical problems. He has no other PMH and seldom sees a physician. He smokes tobacco regularly. His PLT wnl, ESR is 55, Hgb 9, and Albumin 3.1. Coags show prolonged aPTT and normal PT. Adding plasma from a normal subject does not correct the aPTT. What is the cause of the recurrent hemarthroses? A: acquired inhibitor B: factor VIII def C: factor IX def D: secondary syphilis E: vitamin C def

C an elevated aPTT with a normal PT is c/w functional deficiency of Factor VIII, IX, XI, XII, high molecular weight kininogen, or prekallikrein. Congenital or nutritional def of these will be corrected in the lab by addition of serum from a normal subject. The presence of a specific antibody to coat factor is termed an acquired inhibitor. Usually these are directed against Factor VIII, although acquired inhibitors to prothrombin, Factor V, Factor IX, Factor X, and Factor XI are described. Patients with acquired inhibitors are typically older adults (median 60) with pregnancy or postpartum states being less common. No underlying disease is present in 50%. The MC underlying diseases are AID, malignancies (lymphoma, prostate cancer), and dermatologic diseases. Acquired factor VIII or IX inhibitors present clinically in the same fashion as congenital hemophilias. Developing the coagulation disorder later in life is more suggestive of an acquired inhibitor if there is no antecedent hx of coagulopathy. Syphilis infection is a cause of a falsely abnormal aPTT but since this is a lab phenomenon, there is no associated clinical coagulopathy. Vitamin C def may cause gingival bleeding and a perifollicular petechial rash but does not cause significant hemarthroses or a prolonged aPTT. Tobacco hx and labs evidence of chronic illness (anemia, hypoalbuminemia) in this scenario raise the suspicion of underlying malignancy

All but which of the following about lupus anticoagulant (LA) are true? A: lupus anticoagulants typically prolong the aPTT B: a 1:1 mixing study will not correct in the presence of lupus anticoagulants C: bleeding episodes in patients with lupus anticoagulants may be severe and life threatening D: female patients may experience recurrent mid trimester abortions E: lupus anticoagulants may occur in the absence of the other ssx of SLE

C lupus anticoagulants cause prolongation of the coagulation tests by binding to phospholipids. Although most often encountered in patients with SLE, they may develop in normal individuals. The dx is first suggested by prolonged coagulation panels. Failure to correct with incubation with normal plasma confirms the presence of a circulating inhibitor. Contrary to the name, patients with LA activity have normal hemostasis and are not predisposed to bleeding. Instead, they are at risk for venous and arterial thromboembolism. Patients with hx of recurrent unplanned abortions or thrombosis should undergo lifelong A.C. The presence of LA or anticardiolipin antibodies without a hx of thrombosis may be observed as many of these patients will not go on to develop a thrombotic event.

A patient asks about the utility of performing monthly self breast exams. Which statement is correct regarding the utility of and recommendations for SBE? A: reduces mortality only in women undergoing breast biopsy B: most screening societies recommend performing BSE monthly for women <20 years C: self exam leads to increased biopsy rate D: very few breast cancers are first detected by patients E: breast self exam leads to improved survival from breast cancer

C no study of breast self exam has shown a reduced mortality due to breast cancer, despite being a/w higher rates of biopsy. The procedure is still recommended as prudent by many orgs; however, only the ACS recommends BSE in women >19. The USPSTF provides no recommendations and the Canadian excludes its use as a useful screening technique. A substantial fraction of breast cancers are first detected by patients. Though MR have not declined as a result of BSE, the size of lumps being detected by patients have steadily gotten smaller since the 90s.

55 YO F presents with progressive incoordination. PE is remarkable for nystagmus, mild dysarthria, and past-pointing on finger-to-nose testing. She also has unsteady gait. MRI reveals atrophy of both lobes of the cerebellum. Serology reveals anti-Yo antibodies. Which of these is most likely the cause of the syndrome? A: non-small cell lung cancer B: small-cell cancer of the lung C: breast cancer D: NHL E: colon cancer

C one of the better characterized paraneoplastic neurologic syndromes is cerebellar ataxia caused by Purkinje cell drop out in the cerebellum; it is manifested by dysarthria, limb and gait ataxia, and nystagmus. XR imaging reveals cerebellar atrophy. Many antibodies are a/w this syndrome, including anti-Yo, anti-Tr, and antibodies to the glutamate receptor. Although lung cancer, particularly small cell, accounts for a large number of patients with neoplasm-associated cerebellar ataxia, those with this syndrome who display anti-Yo antibodies in the serum typically have breast or ovarian cancer

76 YO M admitted for c/o fatigue for 4 months and fever for the past 1 week. His T has been 38.3C at home. During this time, he has had 5.5-kg weight loss, severe bruising with minimal trauma, and aching sensation in his bones. He last saw his PCP 2 months ago and was dx with anemia of unclear etiology at that time. He has hx of previous L-MCA CVA which has left him with decreased functional status. At baseline, he is able to ambulate in his home with a walker and depends on his caregiver for ADLs. VS show BP 158/86, HR 98, RR 18, SaO2 95%, and T 38C. He appears cachetic with temporal wasting. He has petechiae on hard palate. He has no LAD. On CVS, there is a II/VI systolic ejection murmur. Lungs are clear. Liver is enlarged and palpable 6 cm below RCM. In addition, spleen is also enlarged, about 4 cm below LCM. There are multiple hematomas and petechiae on the extremities. Labs show Hgb 5.1, HCT 15, PLT 12, and WBC 168 with 45% blasts, 30% PMNs, 20% lymphos, and 5% monocytes. Review of PBS confirms AML (M1 subtype, myeloblastic leukemia w/o maturation) with complex chromosomal abnormalities. All of the following confer a poor prognosis for this patient, except: A: advanced age B: complex chromosomal abnormalities C: Hgb <7 D: prolonged interval between ssx onset and diagnosis E: WBC > 100

C patients with acute leukemia frequently present with nonspecific fatigue and weight loss. Additionally, about 50% have ssx for >3 months. Fever is present in only about 10% of patients at presentation, and 5% have abnormal hemostasis. On PE, HSM, sternal tenderness, and evidence of infection or hemorrhage are common. Labs confirm anemia, thrombocytopenia, and leukocytosis. The median WBC is 15. About 20-40% will have <5, another will have >100. Review of the PBS confirms cases. If Auer rods are seen, the diagnosis of AML is certain. Thrombocytopenia (<100) is seen in >75%. Once the diagnosis is confirmed, rapid eval and treatment should be taken. The overall health of the CVS, pulmonary, hepatic, and renal systems should be evaluated as CTX has AE that may cause dysfunction. Among the prognostic factors which predict poor outcomes in AML, age of diagnosis is one of the most important as advanced age groups do not tolerate CTX well. Additionally, advanced age is more likely a/w multiple chromosomal abnormalities that predict poor response to CTX, although some markers predict a better response to CTX. Poor performance status independent of age also decreases survival in AML. Chromosome findings at diagnosis are also very important in predicting outcomes. Responsiveness to CTX and survival are also worse if the WBC >100 or the antecedent course of ssx is prolonged. Anemia, leukopenia, or thrombocytopenia present for >3 months is a poor prognostic factor. However, there is no absolute degree of either which predicts worse outcomes.

34 YO F with PMH of sickle cell anemia presents with 5 days of fatigue, lethargy, and SOB. She denies CP or bone pain. She has had no recent travel. Of note, her 4 year old daughter has had a "cold" 2 weeks before presentation. On exam, she has pale conjunctivae, is anicteric, and is mildly tachycardic. Abdominal exam is wnl. Labs show Hgb 3 (baseline is 8). WBC and PLTs are wnl. Reticulocyte count is undetectable. Total bilirubin is 1.4. LDH is ULN. PBS shows a few sickled cells but a total absence of reticulocytes. The patient is given 2 U PRBCs and admitted. A BM biopsy shows a normal myeloid series but an absence of erythroid precursors. Cytogenetics are normal. What is the best next step? A: make arrangements for exchange transfusions B: tissue typer her siblings for a possible BM transplant C: check parvovirus titers D: start prednisone and cyclosporine E: start broad spectrum abx

C pure red cell aplasia (PRCA) is a condition characterized by the absence of reticulocytes and erythroid precursors. A variety of conditions may cause this. It may be idiopathic. It may be a/w certain medications, such as TMP-SMX and phenytoin. It can e a/w a variety of neoplasms, either as a precursor to heme malignancy such as leukemia or myelodysplasia or as part of an autoimmune phenomenon, as in the case of thymoma. Infections may also cause PRCA. ParvoB19 is a ssDNA virus a/w erythema infectiosum, or fifth disease in children. It is also a/w arthropathy and a flu-like illness in adults. It is thought to attack the P antigen on pro erythroblasts directly. Patients with chronic hemolytic anemia, such as sickle cell patients, or with immunodeficiency are less able to tolerate a transient drop in reticulocytes as their RBCs do not survive in the peripheral blood for adequate times. In this patient, her daughter had an illness before the ssx appeared. It is reasonable to check IgM titers for parvo. If they are positive, a dose of IV-immunoglobulin is indicated. Because her labs and smear are not suggestive of dramatic sickling, an exchange transfusion is not indicated. Immunosuppression with prednisone and/or cyclosporine may be indicated if another etiology of the PRCA is identified. However, this would not be the next step. Similarly, a BM transplant might be a consideration in a young patient with myelodysplasia or leukemia, but there is no evidence of that at this time. ABX have no role in light of her normal WBC and lack of evidence for bacterial infection

22 YO M presents to clinic with a swollen leg. He does not remember any trauma to the leg, but the pain and swelling began 3 weeks ago in the anterior shin area of his L foot. He is a college student and is active in sports daily. XR of the R leg shows a destructive lesion with a "moth-eaten" appearance extending into the soft tissue and a spiculated periosteal reaction. Codman's triangle (a cuff of periosteal bone formation at the margin of the bone and soft tissue mass) is present. What is the most likely dx and optimal therapy? A: chondrosarcoma; CTX curative B: chondrosarcoma; XRT with limited surgical resection C: osteosarcoma; pre-op CTX followed by limb-sparing surgery D: osteosarcoma; XRT E: plasma cell tumor; CTX

C the MC malignant tumors of bone are plasma cell tumors related to myeloma. The bone lesions are lytic due to increase osteoclasts, without osteoblastic new bone formation. Of the nonhematopoietic tumors, the MC are osteosarcomas, chondrosarcomas, Ewings, and malignant fibrous histiocytoma. Osteosarcomas account for 45% of bone sarcomas and produce osteoid (unmineralized bone) or bone. They typically occur in children, adolescents, and adults to the 3rd decade. The "sunburst" appearance of the lesion and Codman's triangle in this young man are indicative of osteosarcoma. They have a predilection for long bones, whereas chondrosarcomas are more often found in flat bones, especially the pelvic and shoulder girdles. Osteosarcomas are radioresistant. Long term survival with combined CTX and surgery is 60-80%. Chondrosarcomas account for 20-25% of bone sarcomas and are MC in adults in 4th to 6th decades. These typically present indolently with pain and swelling. They are often difficult to distinguish from benign bone lesions. Most chondrosarcomas are chemoresistant and the mainstay of therapy is resection of the primary and mets.

You are asked to review the PBS from a patient with anemia. Serum LDH is elevated and there is hemoglobinuria. This patient is likely to have which PE finding? A: goiter B: heme-positive stools C: mechanical second heart sound D: splenomegaly E: thickened calvarium

C this blood smear shows fragmented RBCs of varying size and shape. In the presence of a foreign body within the circulation (prosthetic heart valve, vascular graft), RBCs can become destroyed. Such intravascular hemolysis will also cause serum LDH to be elevated and hemoglobinuria. In isolated extravascular hemolysis, there is no hemoglobin or hemosiderin released into the urine. The characteristic PBS in splenomegaly is Howell-Jolly bodies (nuclear remnants within the RBCs). Certain diseases are a/w extra medullary hematopoiesis (e.g., chronic hemolytic anemias), which can be dated by an enlarged spleen, thickened calvarium, myelofibrosis, or hepatomegaly. The PBS may show tear-drop cells or nucleated RBCs. Hypothyroidism is a/w macrocytosis, which is not demonstrated here. Chronic GI blood loss will cause microcytosis, not schistocytes.

24 YO F presents to the ED c/o red, tender rash that has been spreading across her arms and legs over the past 2 days. She also describes severe diffuse ms pain that is worsened over a week's time. She woke up feeling as though she could not catch her breath and has developed a dry cough over the past several days. She is w/o signifiant medical hx, but recalls that she had similar ssx several years ago, and was told she was having a bad allergic reaction. Her ssx abated with po glucocorticoid taper. She takes no meds, but does take OTC nutritional supplements daily. She cannot describe any allergic trigger to her previous episode or current one. Her FamHx is unremarkable, and her close contacts are not ill. She works in an office, has not pets, and has not travelled internationally. Her labs show WBC 12.1 with total eosinophils of 1.1. Which of the following is the most likely cause? A: early stage of SLE B: gluten allergy C: ingestion of L-tryptophan D: lactose intolerance E: recent viral URTI

C this patient has ssx of eosinophilia-myalgia syndrome, which is triggered by ingestion of contaminants in L-tryptophan-containing products. This is a multi system disease that can present acutely and can be fatal. The two clinical hallmarks are marked eosinophilia and myalgia w/o obvious etiology. Eosinophilic fasciitis, pneumonitis, and myocarditis may be present. Typical eosinophil counts are >1.0. Treatment includes withdrawal of all L-tryptophan containing products and administration of glucocorticoids. Lactose intolerance is very common and typically presents with diarrhea and gas pain temporarily related to ingestion of lactose-containing foods. While SLE can present in a myriad of ways, eosinophilia and myalgia are atypical of this illness. Celiac disease, AKA gluten insensitivity syndrome, is characterized by malabsorption and wt loss and can present with non-GI ssx; but these classically include arthritis and CNS disturbances.

45 YO M is evaluated by PCP for c/o early satiety and wt loss. On PE, his spleen is palpable 10 cm below the left costal margin and is mildly tender to palpation. His labs show WBC 125 (80% PMNs, 9% bands, 3% myelocytes, 3% metamyelocytes, 1% blasts, 1% lymphocytes, 1% eosinophils, and 1% basophils), Hgb 8.4, HCT 27, and PLT 668. BM biopsy demonstrates increased cellularity with increased myeloid to erythroid ratio. Which of the following cytogenetic abnormalities is most likely to be found in this patient? A: deletion of a portion of the long arm of chromosome 5, del(5q) B: inversion of chromosome 16, inv(16) C: reciprocal translocation between chromosome 9 and 22 (Philadelphia chromosome) D: translocations of the long arms of chromosome 15 and 17 E: trisomy 12

C this patient presents with CML, which has an incidence of 1.5 per 100,000 people yearly. The typical age of onset is mid-forties and there is slight male predominance. Half of individuals are asymptomatic at time of diagnosis. If ssx are present, they are typically nonspecific and include fatigue and wt loss. Occasionally patients will have ssx related to splenic enlargement such as early satiety and LUQ pain. Labs suggest CML with high WBC of >100K is typical, with predominant granulocytic ddx, including PMNs, myelocytes, metamyelocytes, and bands. The circulating blast count should be <5%. Anemia and thrombocytosis are common. BM demonstrates nonspecific increase in cellularity with increase in the myeloid to erythroid ratio. The ddx of CML is usually made by demonstrating the presence of the Philadelphia chromosome, a reciprocal translocation between 9:22. This abnormality is present in 90-95% of CML and can be found by FISH or cytogenetics. This translocation results in the fusion of the bcr gene with the abl gene. The bcr-abl fusion protein results in constitutive action of the abl tyrosine kinase enzymes that prevents apoptosis and leads to increased survival of the cells containing the mutation. Ultimately, untreated CML develops into accelerated phase with increasing numbers of mutations and leads to acute blast crisis. The deletion of the long arm of Chromosome 5 is present in some AML and is a/w older age at diagnosis. The inversion of C 16 is typically in AML M4 subtype. The translocation of the long arms of 15 and 17 is the mutation a/w acute promyelocytic anemia that results in arrest of cellular ddx that can be treated with pharmacologic doses of ATRA. Finally, Trisomy 12 is one of several mutations which result in the development of CLL.

52 YO F is evaluated for abdominal swelling with CT that shows ascites and likely peritoneal shedding of tumor, but no other abnormalities. Paracentesis shows adenocarcinoma but cannot be further ddx. A thorough PE, including breast and pelvic exam is wnl. CA-125 levels are elevated. Pelvic US and mammography are normal. Which statement is true? A: compared with other women with known ovarian cancer at a similar stage, this patient can be expected to have less than average survival B: debulking surgery is indicated C: surgical debulking plus cisplatin and paclitaxel is indicated D: bilateral mastectomy and bilateral oophorectomy will improve survival E: fewer than 1% of patients with this disorder will remain disease free 2 years after treatment

C this patient presents with ssx suggestive of ovarian cancer. Although her peritoneal fluid is positive for adenocarcinoma, further speciation cannot be done. Surprisingly, the PE and imaging do not show a primary source. Although the ddx includes gastric cancer or other GIT malignancy and breast cancer, peritoneal carcinomatosis most commonly is 2/2 ovarian cancer in women, even when the ovaries are normal at surgery. Elevated CA-125 levels or the presence of psammoma bodies is further suggestive of an ovarian origin, and such patients should receive surgical debulking and carboplatin or cisplatin plus paclitaxel. Patients with this presentation have a similar stage-specific survival compared to other patients with known ovarian cancer. 10% of patients with this disorder, also known as primary papillary serous carcinoma, will remain disease free 2 years after treatment

all of these have been a/w development of a lymphoid malignancy, except: A: celiac sprue B: H pylori infection C: HBV D: HIV infection E: HHV8 infection F: inherited immunodeficiency syndromes

C: HBV and HCV are common causes of cirrhosis and are strongly a/w development of HCC. HCV, not HBV, can also lead to a lymphoplasmacytic lymphoma, often in the spleen, that resolves with cure of HCV. Other infections are commonly implicated as causes of lymphoma. EBV is a/w a large # of lymphoid malignancies including post transplant lympoproliferative disease (PTLD), Hodgkins, CNS lymphoma, and Burkitt's lymphoma. H pylori is necessary and sufficient for gastric MALTOMA, and cure can be achieved with eradication of the organisms in many cases. HHV8 is a known cause of body cavity lymphoma, including primary pleural lymphoma. Celiac sprue is a/w GIT lymphoma. Many collagen vascular disorders and their treatments (TNF alpha inhibitors) have also been a/w lymphomas, as have acquired and inherited immunodeficiencies

All of the following statements regarding the epidemiology and RF for AML are true, except: A: anticancer drugs such as alkylating agents and topo-II inhibitors are the leading cause of drug-associated myeloid leukemia B: individuals exposed to high dose XRT are at risk for AML whereas individuals treated with therapeutic radiation are not unless also given alkylating agents C: men have a higher incidence of AML than women D: the incidence of AML is greatest in individuals <20 YO E: trisomy 21 is a/w an increased risk of AML

D AML are a group of heme malignancies derived from heme stem cells which have acquired chromosomal mutations that ppx ddx into mature myeloid cells. The specific abnormalities predict in which stage of ddx the cell is arrested and are a/w several subtypes of AML that have been ID'd. In the US, >16,000 new cases of AML are diagnosed yearly, and the numbers of new cases have increased in the past 10 years. Men are diagnosed more frequently than women. In addition, older age is a/w increased incidence of AML. AML is uncommon in adolescents. Other known RF include hereditary genetic abnormalities, XRT and chemical exposures, and drugs. The most common hereditary link is trisomy 21. Others a/w increased risk include defective DNA repair (Fanconi) and Ataxia telangiectasia. Survivors of the atomic bomb explosion in Japan have a higher incidence of AML as have survivors of other high dose radiation exposures. However, therapeutic XRT is not a/w increased risk of AML unless the patient was also concomitantly given alkylating agents. Anticancer drugs are the MC causes of drug-associated AML. Of the CTX, alkylating agents and topo-II inhibitors are the drugs most likely a/w AML

All of these are suggestive of Fe def anemia, except: A: koilonychia B: pica C: decreased serum ferritin D: decreased TIBC E: low reticulocyte response

D Fe def anemia is a condition in which there is anemia and clear evidence of Fe def. Initially, a state of negative Fe balance occurs during which Fe stores becomes slowly depleted. Serum ferritin may decrease, and the presence of stainable Fe on BM prep decreases. When Fe stores deplete, serum Fe begins to fall. TIBC starts to increase, reflecting the presence of unbound transferrin. Once the transferrin sat falls to 15-20%, Hgb synthesis is impaired. The PBS reveals the presence of microcytic and hypo chromic RBCs. Reticulocytes may also become hypo chromic. Reticulocyte numbers are reduced relative to the level of anemia, reflecting a hypoproductive anemia 2/2 Fe def. Clinically, these patients exhibit the usual signs of anemia: fatigue, pallor, and reduced exercise capacity. Cheilosis and koilonychia are signs of advanced tissue Fe def. Some patients have pica, a desire to ingest certain materials, such as ice (pagophagia) and clay (geophagia)

all of the enzyme deficiencies that lead to porphyrias are inherited either as AD or AR traits with one exception. Which of the following most commonly occurs sporadically? A: 5-ALA dehydratase-deficient porphyria B: acute intermittent porphyria C: erythropoietic porphyria D: porphyria cutanea tarda E: variegate porphyria

D PCT is the most common porphyria and usually arises from a sporadic gene mutation leading to a deficiency of the enzyme uroporphyrinogen decarboxylase. Despite being a hepatic porphyria, it usually presents with blistering skin lesions on the back of the hands. These lesions can lead to areas of atrophy and scarring after resolution. Neurologic ssx are absent. Any condition that increases hepatic iron will exacerbate PCT. PCT is a/w ETOH abuse, HCV, estrogens, elevated serum Fe, and HIV. Rare forms of PCT are familial, 5-ALA dehydratase-deficient porphyria is inherited in AR fashion. Acute intermittent porphyria, erythropoietic porphyria, and variegate porphyria are AD.

Aplastic anemia has been a/w all of these, except: A: carbamazepine therapy B: methimazole therapy C: NSAIDs D: parvo B19 infection E: seronegative hepatitis

D aplastic anemia is pancytopenia with BM hypocellularity. Aplastic anemia may be acquired, iatrogenic (CTX), or genetic (e.g., Fanconi). Acquired aplastic anemia may be due to drugs or chemicals (expected toxicity or idiosyncratic effects), viral infection, immune disease, PNH, pregnancy, or idiopathic causes. Aplastic anemia from idiosyncratic drug reactions (including those listed as well others including as quinacrine, phenytoin, sulfonamides, cimetidine) are uncommon but may be encountered given the wide usage of some of these agents. In these cases there is usually not a dose-dependent response; the reaction is idiosyncratic. Seronegative HEP is a cause of aplastic anemia, particularly in young men who recovered from an episode of liver inflammation 1-2 months prior. ParvoB19 most commonly causes pure red cell aplasia, particularly in patients with chronic hemolytic states and high RBC turnover (e.g., SCD)

Which of these statements about cardiac toxicity from cancer treatments is true? A: doxorubicin-based cardiac toxicity is idiosyncratic and dose-independent B: anthracycline-induced CHF is reversible with time and control of RF C: mediastinal irradiation often results in acute pericarditis during the first few weeks of treatment D: chronic constrictive pericarditis often manifests symptomatically up to 10 years after treatment E; the incidence of coronary atherosclerosis in patients who have a hx of mediastinal irradiation is the same as that in age-matched controls

D cancer is the 2nd lead cause of mortality in the US. Millions who are alive today have PMH of cancer. Cardiac toxicity is typically related to the prior treatment with anthracycline-based CTX or mediastinal irradiation. This is MC seen in patients who survived Hodgkins or NHL. Anthracycline-related cardiotoxicity is dose-dependent. About 5% of patients who receive more than 550 mg/m2 of doxorubicin will develop CHF. Rates are higher in those with other cardiac RF and those who have received mediastinal irradiation. Unfortunately, anthracycline-related CHF is typically not reversible. Intracellular chelators or liposomal formulations of the CTX may prevent cardiotoxicity, but their impact on cure rates is unclear. Radiation has both acute and chronic effects on the heart. It may result in acute and chronic pericarditis, myocardial fibrosis, and accelerated atherosclerosis. The mean time to onset of "acute" pericarditis is 9 months after treatment, and so caretakers must be vigilant. Similarly, chronic pericarditis may manifest years later.

31 YO F referred for evaluation of anemia. She describes 2-month hx of fatigue. She denies aBD pain, but notes that her abdomen has become slightly more distended in recent weeks. PMH is otherwise stable. The patient's parents are alive, and she has 3 healthy siblings. PE is significant for pale conjunctiva and a palpable spleen 4cm below the LCM. HCT is 31 and bilirubin is normal. The reticulocyte % is low. Haptoglobin and LDH are normal. PBS shows numerous teardrop-shaped RBCs, nucleated RBCs, and occasional myelocytes. A BM aspirate is unsuccessful, but a bx shows hyper cellular BM with trilineage hyperplasia and findings c/w presumed dx of chronic idiopathic myelofibrosis. You transfuse her to HCT 40. What is the most appropriate next step? A: give EPO B: follow up in 6 months C: institute combined-modality CTx D: perform HLA match of her siblings E: perform a splenectomy

D chronic idiopathic myelofibrosis is a clonal disorder of multipotent hematopoietic progenitor cell of unknown etiology that is characterized by a marrow fibrosis, myeloid metaplasia, extra medullary hematopoiesis, and splenomegaly. The PBS shows features of extra medullary hematopoiesis with teardrop shaped RBCs, immature myeloid cells, and abnormal PLTs. WBCs and PLTs may both be elevated. The median survival is poor at only 5 years. These patients eventually succumb to increasing organomegaly, infection, and possible transformation to acute leukemia. There is no specific therapy for chronic idiopathic myelofibrosis. EPO has not shown to be consistently effective and may exacerbate splenomegaly. Supportive care with transfusions is necessary as anemia worsens. CTx has no role in changing the natural history of the disease. Some newer agents, IFN and thalidomide, may play a role, but their place is not clear. Splenectomy may be necessary in symptomatic patients with massive splenomegaly. However, extra medullary hematopoiesis may worsen with rebound thrombocytosis and compensatory hepatomegaly. The only potential curative modality is allogenic BM transplant. M/M are high, particularly in older patients. In light of this patient's young age and the presence of 3 healthy siblings, HLA matching is the most reasonable step.

the triad of portal venous thrombosis, hemolysis, and pancytopenia suggests which of the following? A: acute promyelocytic leukemia B: HUS C: leptospirosis D: PNH E: TTP

D each of these has a rather characteristic set of findings that are virtually diagnostic for the disease once it has progressed to severe stage. Both HUS and TTP cause hemolysis and thrombocytopenia, as well as fevers. CVA and AMS may occur more commonly in TTP, and renal failure more so in HUS. Severe leptospirosis, or Weil's disease, is notable for fevers, hyperbili, and renal failure. Conjunctival suffusion is another helpful clue. APML is notable for anemia, thrombocytopenia, and either elevated or decreased WBC, all in the presence of DIC. PNH is a rare disorder characterized by hemolytic anemia (particularly at night), venous thrombosis, and deficiency hematopoiesis. It is a stem cell-derived intracorpuscular defect. Anemia is usually moderate in severity, and is often a/w concomitant granulocytopenia and thrombocytopenia. Venous thrombosis occurs MC than in the population at large. The intraabdominal veins are often involved, and patients may present with Budd-Chiari. Cerebral sinus thrombosis is a common cause of death in patients with PNH. The presence of pancytopenia and hemolysis should raise suspicions for this diagnosis, even before the development of a venous thrombosis. In the past PNH was diagnosed by abnormalities on the Ham or sucrose lysis test; however, current flow cytometry analysis of GPI-linked proteins such as CD55 and CD59 on RBC and granulocytes is recommended.

Which of the following tumor characteristics confers a poor prognosis in patients with breast cancer? A: ER+ B: good nuclear grade C: low proportion of cells in S phase D: over expression of HER2 E: PR+

D pathologic staging remains the most important determination of prognosis overall. Other factors have an impact on survival and the choice of therapy. Tumors that lack ER and/or PR are more likely too recur. The presence of ER, particularly in postmenopausal women, is also an important factor in determining adjuvant CTX. Tumors with a high growth rate are a/w early relapse. Measurement of the proportion cells in S-phase is a measure of the growth rate. Tumors with more than the median number of cells in S-phase have a higher risk of relapse and improved response rate to CTX. Histologically, tumors with poor nuclear grade have a higher risk of recurrence than do tumors with a good nuclear grade. At the molecular level, tumors that overexpresses HER2 or have mutated p53 gene portend a poorer prognosis for patients. The over expressed erbB2 protein is also useful in designing optimal treatment regimens, and a human monoclonal antibody to erbB2 (Herceptin) has been developed

All of the following are a/w pure red cell aplasia, except: A: anterior mediastinal masses B: CT disorders C: giant pronormoblasts D: low EPO levels E: parvo b19 infection

D pure red cell aplasia is a normochormic, normocytic anemia with absent erythroblasts on the BM, hence the diminished number or lack of reticulocytes. The BM shows red cell aplasia and the presence of giant pronormoblasts. Several conditions have been a/w pure red cell aplasia, including viral infections such as ParvoB19 (which can have cytopathic BM changes), HIV, EBV, HTLV, and HBV; malignancies such as thymomas and lymphoma (which often present with anterior mediastinal masses); CT disorders such as SLE and RA; pregnancy; drugs; and hereditary disorders. EPO levels are elevated because of anemia

Which source of stem cell is INCORRECTLY paired with the challenge a/w their clinical application? A: BM mesenchymal SC: transplanted cell may not ddx into the desired type B: embryonic SC: high potential to form teratomas C: organ-specific multipoint SC: difficult to isolate from tissues other than BM D: umbilical cord blood stem cells: GVHD

D regardless of the source of stem cells used, a number of generic and specific problems must be overcome before successful clinical applications are available. Embryonic cells tend to develop abnormal karyotypes and have potential to form teratomas. Umbilical cord blood stem cells have less GVHD than marrow-derived and are less likely to be contaminated by herpes virus. Organ-specific multipotent stem cells are easy to isolate from the marrow but are difficult to isolate from tissues such as heart and brain. Early studies of BM mesenchymal cells have shown that transplanted cells fuse with cells resident in the organ

All of these cause prolongation of the aPTT that does not correct with a 1:1 mixture with pooled plasma, except: A: lupus anticoagulant B: factor VIII inhibitor C: heparin D: factor VII inhibitor E: factor IX inhibitor

D the aPTT involves the factors of the intrinsic pathway of coagulation. Prolongation of aPTT reflects either a deficiency of one of these factors (VIII, IX, XI, XII, etc) or inhibition of the activity of one of the factors or components of the aPTT assay (i.e., phospholipids). This may be further characterized by the "mixing study", in which the patient's plasma is mixed with pooled plasma. Correction of the aPTT reflects a deficiency of factors that are replaced by the pooled sample. Failure to correct the aPTT reflects the presence of a factor inhibitor or phospholipid inhibitor. Common causes of a failure to correct include the presence of heparin in the sample, factor inhibitors (VIII inhibitor being the MC), and the presence of antiphospholipid antibodies. Factor VII is involved in the extrinsic pathway. Inhibitors to factor VII would prolong the pT.

48 O M referred by acute care center because of a nodule on CXR. 3 weeks ago, he was diagnosed with PNA after reporting 3 days of fever, cough, and sputum production. The CXR shows a small, RLL alveolar infiltrate and LUL 1.5 cm round nodule. He was treated with abx and is now asymptomatic. A repeat CXR shows that the RLL PNA is resolved, but the nodule is still present. He is asymptomatic. He smoked 1 ppd for 25 years and quit 3 years ago. He never had a prior CXR. CT scan shows that the nodule is 1.5 x 1.7 cm and is located centrally in the LUL, has no calcifications, and has slightly scalloped edges. There is no mediastinal adenopathy or pleural effusion. Which is the best next step? A: bronchoscopy B: mediastinoscopy C: MRI D: FDG PET scan E: repeat chest CT in 6 months

D the evaluation of a solitary pulmonary nodule (SPN) remains a combination of art and science. Approximately 50% of SPNs which are less than 3 cm turn out to be malignant, but the studies have found a range between 10-70% on patient selection. If the SPN is malignant, surgical therapy can result in 80% 5-year survival. Most benign lesions are infectious granulomas. Spiculted or scalloped lesions are more likely to be malignant, whereas lesions with central or popcorn calcifications are more likely to be benign. Masses (>3 cm) are usually malignant. PET scan has added a new test to the options for evaluation. PET has over 95% sensitivity and 75% specificity for ID a malignant SPN. False negatives occur with small (< 1 cm) tumors, bronchoalveolar carcinomas, and carcinoid tumors. False positives are usually due to inflammation. In this patient with moderate malignancy risk (age >45, lesion >1 cm, + smoking hx, suspicious lesion, no prior XR) a PET scan would be the most reasonable choice. PET is also useful for staging. The diagnostic accuracy of PET for malignant mediastinal LN approaches 90%. Another option would be a transthoracic needle biopsy, with sensitivity of 80-95% and specificity 50-85%. Transthoracic needle aspiration has the best results and fewest cx (pneumothorax) with peripheral lesions versus central lesions. Bronchoscopy has very poor yield for lesions <2 cm. Mediastinoscopy would be of little value unless PET or CT raised a suspicion of nodal disease. MRI will not add any info and is less able than CT to visualize lesions in the lung parenchyma. Repeat chest CT is a reasonable option for a patient with low clinical suspicion.

Which of the following pairs of CTX and complication is incorrect? A: daunorubicin - CHF B: bleomycin - interstitial fibrosis C: cyclophosphamide - hematuria D: cisplatin - liver failure E: ifosfamide - Fanconi syndrome

D the focus of cancer care is cure. Many individuals who are fortunate enough to survive malignancy will nevertheless bear chronic stigmata, both psychological and medical, of the treatment. Anthracyclines, which are frequently used for breast cancer, Hodgkins, lymphoma, and leukemia are toxic to myocardium and, at high doses, can lead to heart failure. Bleomycin results in pulmonary toxicity. Pulmonary fibrosis and pulmonary venooclusive disease may result. Liver dysfunction is common with a number of CTX. However, cisplatin primarily causes renal toxicity and AKI. It may also cause neuropathy and hearing loss, but liver dysfunction is not a common cx. Ifosfamide may cause neurologic toxicity and renal failure. Also, it may cause proximal tubule defect resembling Fanconi syndrome. Cyclophosphamide may result in cystitis and increase the long term risk of bladder cancer. Administration of mesna ameliorates but does not completely eliminate the risk.

23 YO M presents with diffuse bruising. He otherwise feels well. He takes no meds, does not use supplements, and does not use IVD. His PMH is negative for prior illnesses. He is a college student and works as a barista in a coffee shop. A CBC shows ANC 780, HCT 18, and PLT 21. BM biopsy reveals hypocellular with fatty marrow. Chromosome studies of PBS and BM cells are performed which exclude Fanconi anemia and MDS. The patient has a fully histocompatible brother. Which is the best therapy? A: anti-thymocyte globuli plus cyclosporine B: glucocorticoids C: growth factors D: hematopoietic SCT E: RBC and PLT transfusion

D this patient has aplastic anemia. In the absence of drugs or toxins that cause BM suppression, it is most likely that he has immune-mediated injury. Growth factors are not effective in the setting of hypo plastic marrow. Transfusion should be avoided unless emergently needed to prevent the development of alloantibodies. GC have no efficacy in aplastic anemia. Immunosuppresion with anti-thymocyte globulin and cyclosporine is a therapy with proven efficacy for this AID with a response rate up to 70%. Relapses are common and MDS or leukemia may occur in approx 15% of treated patients. Immunosuppression is the treatment of choice for patients w/o suitable BMT donors. BMT is the best current therapy for young patients with matched sibling donors. Allogenic BMT from matched siblings result in long term survival in >80% patients, with better results in children than adults

You are evaluating a 45 YO M with acute UGIB in the ED. He reports increasing abdominal girth over the past 3 months a/w fatigue and anorexia. He has not noticed any LE edema. His PMH is significant for Hemophilia A diagnosed as a child with recurrent elbow hemarthroses in the past. He has been receiving infusions of FVIII for most of his life, and received his last injection earlier that day. His BP 85/45 with HR 115. His ABD exam is tense with a positive fluid wave. HCT 21. Renal function and UA is normal. His aPTT is minimally prolonged, his INR is 2.7, PLTs are normal. Which of the following is most likely to yield a dx for the cause of his GI bleeding? A: Factor VIII activity level B: H pylori antibody test C: HBsAg D: HCV RNA E: mesenteric angiogram

D this patient presents with a significant UGIB with prolonged PT. Hemophilia should not cause a prolonged PT. This and the presence of ascites raise the possibility of liver disease and cirrhosis. The contamination of blood products in the 70s and 80s resulted in widespread transmission of HIV and HCV within the hemophilia population receiving factor infusions. It is estimated in 2006 that >80% of hemophilia patients >20 YO are infected with HCV. Viral inactivation steps were introduced in the 80s and recombinant F VIII and IX were first produced in the 90s. HCV is the major cause of morbidity and the second leading cause of death in patients exposed to older factor concentrates. Patients develop cirrhosis and the complications including ascites and variceal bleeding. ESLD requiring a liver transplant will be curative for the cirrhosis and the hemophilia (the liver produces FVIII). HBV was not transmitted in significant numbers to patients with hemophilia. Diverticular disease or PUD would not explain the prolonged PT. Patients with inadequately repleted FVIII levels are more likely to develop hemarthroses than GI bleeds and the slightly prolonged apTT makes this unlikely

Which of these carries the best disease prognosis with appropriate treatment? A: Burkitt's lymphoma B: Diffuse large B cell lymphoma C: Follicular lymphoma D: Mantle cell lymphoma E: Nodular sclerosing Hodgkin's disease

E Classical Hodgkins disease carries a better prognosis than all types of NHL. Patients with good prognostic factors can achieve cure with extended field radiation alone, while those with higher risk disease often achieve cure with high-dose CTX and sometimes rad. The chance of cure is so high (>90%) that many protocols are now considering longterm sequelae of current therapy such as carcinomas, hypothyroidism, premature CAD, and constrictive pericarditis in those receiving RT. Combination CTX with ABVD appears to be the form of treatment with the lowest risk of late fatal complications.

which of the following best describes the action of Clopidogrel? A: activated Antithrombin and inhibits clotting enzymes B: binds to the activated GpIIb/IIIa receptor on the PLT surface to block binding of adhesive molecules C: inhibits COX-1 on PLTs to decrease production of TXA2 D: inhibits PDE to block the breakdown of cAMP to inhibit PLT activation E: irreversibly binds P2Y12 to prevent ADP-induced PLT aggregation

E Clopidogrel and Ticlopidine are the (2) currently available members of the thienopyridine class of antipLT agents. As demonstrated in the figure below, the MOA of these agents is to prevent ADP-induced PLT aggregation by irreversibly binding and inhibiting P2Y12 receptor. Both agents are prodrugs that require hepatic activation by P450 system; in the usual dose they require several days to reach maximal effectiveness. Clopidogrel is a more potent agent than Ticlopidine with fewer AE, and thus has replaced it in clinical practice. Other antiPLT drugs act at other sites in the cascade that leads to PLT aggregation. ASA is the MC used antiPLT agent. At the usual doses, ASA inhibits COX1 to prevent production of TXA2, a potent PLT agonist. Dipyridamole is a weak PLT inhibitor alone and acts as a PDE inhibitor. In addition, dipyridamole blocks the uptake of adenosine by PLTs. When combined with ASA, dipyridamole has been shown to decrease the risk of CVA, but because it acts as a vasodilator, there is concern that it might increase the risk of CVD events in severe CAD. A final class of antiPLT agents is the glycoprotein IIb/IIIa inhibitors, which include Abciximab, eptifibatide, and tirofiban. Each of these has a slightly different site of action, but all decrease the ability of PLTs to bind adhesive molecules such as fibrinogen and vWF. Thus, these agents decrease PLT aggregation. ABCiximab is a monoclonal directed against the activated form of GpIIb/IIIa. Tirofiban and eptifibatide are small synthetic molecules which bind to various sites of the GpIIb/IIIa receptor to decrease PLT aggregation.

31 YO M with Hemophilia A is admitted with persistent gross hematuria. He denies recent trauma or any hx of genitourinary pathology. The exam is unremarkable. HCT is 28. All of the following are treatments for hemophilia A, except: A: desmopressin (ddAVP) B: fresh frozen plasma (FFP) C: cryoprecipitate D: recombinant factor VIII E: plasmapheresis

E Hemophilia A results from a deficiency of factor VIII. Replacement of FVIII is the centerpiece of treatment. Cessation of ASA or NSAIDs is highly recommended. FFP contains pooled plasma from human sources. Cryoppt refers to FFP that is cooled, resulting in the ppt of material at the bottom of the plasma. This product contains about half of the FVIII activity of FFP in 1/10 of the volume. Both are therefore reasonable treatment options. DDAVP causes the release of a number of factors and VWF from the liver and endothelial cells. This may be useful for patients with mild hemophilia. Recombinant or purified factor VIII (i.e., Humate P) is indicated in patients with more severe bleeding. Therapy may be required for weeks, with levels of factor VIII kept at 50%, for post-surgical or severe bleeding. Plasmapheresis has no role in the treatment of Hemophilia A.

38 YO F referred for evaluation of an elevated H/H that was discovered during an evaluation of recurrent HA. Until 8 months ago, she was in good health, but developed increasingly persistent HA with intermittent vertigo and tinnitus. She was originally RX sumatriptan for presumed migraine HA, but did not experience relief of ssx. A CT scan showed no evidence of mass lesion. During evaluation of her HA, she was found to have a Hgb 17.3, and HCT 52. Her only other ssx is diffuse itching after hot showers. She is a nonsmoker. She has no hx of pulmonary or cardiac disease. On PE, she appears well. BMI 22, VS shows BP 148/84, HR 86, RR 12, Sa 99% on RA. She is afebrile. The PE including full neuro is wnl. There are no heart murmurs. There is no splenomegaly. Peripheral pulses are wnl. Labs confirm elevated H/H. She also has a PLT count 650, WBC 12.6 with normal ddx. Which of the following tests should be performed next? A: BM biopsy B: EPO level C: Genetic testing for JAK2 V617F mutation D: Leukocyte ALP E: RBC mass and plasma volume determination

E In a patient with elevated H/H, the initial step is determine whether erythrocytosis represents a true elevation in RBC mass or whether spurious erythrocytosis is present due to plasma volume contraction. This step may not be necessary however in those individuals with Hgb > 20. Once absolute erythrocytosis has been determined by measurement of RBC mass and plasma volume, the cause must be determined. If there is not an obvious cause of the erythrocytosis, an EPO should be checked. An elevated EPO suggests hypoxia or autonomous production of EPO as the cause. However, a normal EPO does not exclude hypoxia as a cause. A low EPO should be seen in PV, the most likely cause of erythrocytosis here. PV is often discovered incidentally when elevated Hgb is found during testing for other reasons. When ssx are present, the MC complaints are related to hyper viscosity of the blood and include vertigo, HA, tinnitus and TIA. Patients may also complain of pruritus after showering. Erythromelalgia is the term given to the ssx complex of burning, pain, and erythema in the extremities and is a/w thrombocytosis in PV. Isolated SBP HTN and splenomegaly may be found. In addition to elevated RBC mass and low EPO, other labs include thrombocytosis and leukocytosis with abnormal WBCs present. Uric acid levels and leukocyte ALP may be elevated, but are not diagnostic for PV. Approximately 30% of individuals with PV are homozygous for Jak2, and over 90% are heterozygous. This mutation on the short arm of Chromosome 9 causes constitutive activation of Jak2 protein, a tyrosine kinase which renders RBCs resistant to apoptosis and allows them to continue production independently from EPO. However, not every patient with PV expresses this mutation. Thus, it is not recommended as a diagnostic test for PV at this time. BM biopsy provides no specific information in PV and is not recommended

34 YO F presents for LLE swelling and pain. She is obese and 8 weeks postpartum. She recently traveled 6h by airplane to visit her parents with her infant. She has no SOB, palpitations, or syncope. She is currently on no meds, except iron tablets. She is otherwise healthy. Her VS: HR 86, BP 110/80, T 37, and RR 12. Wt is 98 kg and Ht 170 cm. The LLE is swollen, tender, and warm to touch. A Homan's sign is +, but there are no palpable cords. A LE Doppler shows a thrombosis in the common and superficial femoral veins of the left leg. You are considering outpatient treatment with enoxaparin. All of the following are true about LMWH, except: A: in patients with uncomplicated DVT, LMWH is a safe and effective alternative to IV-heparin and is a/w reduced health care costs compared to IV heparin B: LMWH can be safely used in pregnancy, but factor Xa should be monitored to ensure adequate AC C: monitoring of factor Xa levels is unnecessary in most patients as there is a predictable dose-dependent AC effect D: there is a decrease in the risk of development of heparin-induced thrombocytopenia with use of LMWH E: this patient's recent pregnancy is a CONTRA for the use of LMWH, because there is a greater risk of bleeding with LMWH compared to IV-heparin

E LMWH have become widely used in the management of uncomplicated DVT and PE due to their ease of administration and predictable effects. LMWH is derived from unfractionated heparin by chemical or enzymatic depolymerization that results in smaller fragments of heparin, weighing approx 1/3 the mean molecular mass of UFH. The MOA is different from that of heparin in that the AC effect of LMWH is related to its ability to potentiate factor Xa inhibition via activating anti-thrombin. While heparin does have this ability, it acts primarily as a cofactor to activate anti-thrombin, and binding antithrombin to thrombin. IN order to activate antithrombin, an 18U polysaccharide chain is required. With a mean mass of 5,000 kd, the average pentasaccharide chain of LMWH is only 17U, and thus over half the LMWH molecules lack the ability to bridge antithrombin to thrombin. A further difference is that it is less bound to proteins in plasma, resulting in >90% bioavailability after SQ injection. Thus, it has a more predictable AC response and a longer half-life. Because of its kinetics, most do not require monitoring of Factor Xa levels to ensure adequate AC, allowing for outpatient treatment of uncomplicated DVT and PE. When outcomes are compared with heparin, LMWHs are equally effective, but there is substantial health care savings when outpatient treatment is used. Furthermore, studies show that serious bleeding events are less likely to occur with LMWH than with UFH. Thrombocytopenia is also less likely. A meta-analysis of 5275 patients on 13 studies suggests that the rate of thrombocytopenia in patients on UFH and LMWH may actually be similar. Caution should be taken, however, when using LMWH in individuals who are obese, pregnant, or have renal insufficiency. In these instances, monitoring Factor Xa is required to ensure adequacy of dosing w/o evidence of drug accumulation

A patient with longstanding HIV infection, .ETOH abuse, and asthma is seen in the ED for 1-2 days of severe wheezing. HE has not been taking medications for months. He is admitted to the hospital and treated with nebulizer therapy and systemic CS. His CD4 is 8 and viral load > 750,000. His WBC is 3.2 with 90% PMNs. He is accepted into an inpatient substance abuse rehabilitation program and before d/c is started on opportunistic infection pox, bronchodilators, a prednisone taper over 2 weeks, ranitidine, and HART. The rehab pages you 2 weeks later; a routine lab check shows WBC 900 with 5% PMNs. Which new drug explains this? A: darunavir B: efavirenz C: ranitidine D: prednisone E: TMP-SMX

E Many drugs can lead to neutropenia, MC via retarding PMN production in the BM. Of the list shown, TMP-SMX is the most likely culprit. Other common causes include: 1. Alkylating agents (cyclophosphamide, busulfan) 2. Antimetabolites (methotrexate, 5FU) 3. PCN 4. Sulfa drugs 5. Anti-thyroid drugs 6. Anti-psychotics 7. Anti-inflammatory Prednisone, when used systemically, often causes an increase in circulating PMN count, as it leads to demmargination of PMNs and BM stimulation. Ranitidine, H2RA, is well-described cause of thrombocytopenia but has not been implicated in neutropenia. Efavirenz is a NNRTI, whose main AE include a morbilliform rash and CNS effects like strange dreams and confusion. The presence of these six does not require drug cessation. Darunavir is a new protease inhibitor that is well tolerated. Common AE include a maculopapular rash and lipodystrophy, a class effect for all protease inhibitors.

Which of these labs is most useful for predicting return of renal function in a patient with tumor lysis syndrome and AKI? A: creatinine B: phosphate C: potassium D: serum pH E: uric acid

E TLS is characterized by hyperuricemia, hyperkalemia, hyperphosphatemia, and hypocalcemia. Metabolic acidosis occurs frequently. AKI is common, and HD should be considered early in the treatment of the problem. Effective cancer therapy kills cells, which release uric acid from the turnover of nucleic acids. In an acidic environment, uric acid can ppt in the renal tubules, medulla, and collecting ducts leading to AKI. Hyperphosphatemia produces a reciprocal depression in the serum calcium. Indications for HD include extreme hyperkalemia (>6), hyperuricemia (>10), hyperphosphatemia (>10 or rapidly increasing), or symptomatic hypocalcemia. Daily uric acid levels should be monitored; excellent renal recovery can be expected once the uric acid is <10

45 YO M is evaluated by PCP for c/o early satiety and wt loss. On PE, his spleen is palpable 10 cm below the left costal margin and is mildly tender to palpation. His labs show WBC 125 (80% PMNs, 9% bands, 3% myelocytes, 3% metamyelocytes, 1% blasts, 1% lymphocytes, 1% eosinophils, and 1% basophils), Hgb 8.4, HCT 27, and PLT 668. BM biopsy demonstrates increased cellularity with increased myeloid to erythroid ratio. Which of the following cytogenetic abnormalities is most likely to be found in this patient? A: deletion of a portion of the long arm of chromosome 5, del(5q) B: inversion of chromosome 16, inv(16) C: reciprocal translocation between chromosome 9 and 22 (Philadelphia chromosome) D: translocations of the long arms of chromosome 15 and 17 E: trisomy 21

E a small proportion of cancers occur in patients with a. genetic predisposition. Roughly 100 syndromes of familial cancer have been reported. Recognition allows for genetic counseling and increased cancer surveillance. Down's syndrome (trisomy 21) is characterized clinically by a variety of features, including moderate-severe learning disability, a facial and MSK deformities, duodenal atresia, congenital heart disease, and increased risk of leukemia. Fanconi anemia is a condition that is a/w defects in DNA repair. There is higher incidence of cancer, with leukemia and myelodysplasia being the MC cancers. VHL syndrome is a/w hemangioblastoma, renal cysts, pancreatic cysts and carcinomas, and renal cell cancer. NF Type I and II are both a/w increased tumor formation. NF II is more a/w schwannoma. Both carry a risk of malignant peripheral nerve sheath tumor. Fragile X is a condition a/w chromosomal instability of the X chromosome. These patients have MR, typical morphologic features including macroorchidism and prognathic, behavior problems, and seizures. Increased cancer incidence has not been described

Which of the following ssx is most suggestive of an esophageal mass? A: early satiety B: liquid phase dysphagia only C: odynophagia with CP D: oropharyngeal dysphagia E: solid phase dysphagia progressing to liquid phase dysphagia

E although esophageal masses and cancer can lead to several types of dysphagia, the MC c/o solid food dysphagia that worsens to a point that liquids are also hard to swallow. Such a complaint warrants upper endoscopy, particularly if the patients falls in a high-risk group for esophageal cancer, with careful exam of the stomach, trachea, and larynx. Odynophagia with CP is more reminiscent of ulcerative disease of the esophagus due to either infection, such as CMV or Candida, or pill esophagitis. Spasms causes severe pain as well, but this may occur independent of swallowing. Liquid phase dysphagia often implies a functional disorder of the esophagus rather than a mass-like obstruction. A barium swallow or cineesophagram in conjunction with a thorough HP exam may prove diagnostic. Oropharyngeal dysphagia usually localizes disease quite specifically to the oropharynx. Early satiety is often due to gastric obstructions or extrinsic compression of the stomach (splenomegaly is common), or to a functional gastric disorders such as gastroparesis.

the MC cause of high serum Ca in a patient with known cancer is: A: ectopic production of PTH B: direct destruction of bone by tumor cells C: local production of TNF and IL-6 by bony mets D: high levels of 1,25-OH Vitamin D E: production of PTH-r protein

E although it was once thought that most cases of hyperCa of malignancy are due to a direct resorption of bone by the tumor, it is now recognized that 80% of such instances occur because of the production of a protein called PTHrP. This protein shares 80% homology in the first 13 terminal AA with native PTH. The aberrantly produced molecule is essentially functionally identical to native PTH in that it causes renal Ca conservation, osteoclast activation with bony resorption, renal phosphate wasting, and increased levels of of urinary cyclic adenine monophosphate (AMP). Only about 20% of cases of the hypercalcemia of malignancy are due to local production of substances, such as TGF, IL-1 or IL-6, which causes bone resorption at the local level and release of Ca from bony stores. Although aggressive hydration with saline and administration of LOOPs are helpful in the short-term management of patients with the hypercalcemia, the most important therapy is a bisphosphonate, such as Pamidronate, that will control the labs and the associated ssx in the vast majority of patients. SSX are nonspecific and include fatigue, lethargy, polyuria, n/v, and decreased mental acuity.

59 YO M admitted with a painful, blistering rash on the dorsal aspects of both hands. He has a PMH of ETOH abuse and admits to a recent relapse and has been drinking heavily over the past week. He is admitted and stabilized. A dx of PCT is made based on increased circulating porphyrins in the blood and decreased uro-decarboxylase activity. He is d/c'd to rehab and follows up 2 weeks later. He has been abstinent from ETOH but his rash persists, and now he has some blistering on his legs and feet. Which of the following treatments is most appropriate? A: hydroxyurea B: IV Fe infusion weekly while following serum Fe levels C: po Fe plus Vitamin C D: the rash of PCT can take months to resolve; the patient should continue to abstain from ETOH and be followed closely E: weekly phlebotomy until ferritin normalizes

E any increase in hepatic Fe will exacerbate PCT, and effort should be made to minimize Fe overload. The first step in mgmt of PCT is to ID and d/c potential triggers (ETOH, estrogens, Fe supplements). PCT that does not respond to these conservative measures requires weekly phlebotomy with the goal of reducing hepatic Fe. In the above case, conservative measures have not led to remission and phlebotomy is necessary. Serum ferritin can be used as a gauge of hepatic Fe overload and should guide the course of phlebotomy. Fe infusion or po iron would exacerbate PCT by increasing Fe stores. Hydroxyurea is used to treat SCD and some forms of essential thrombocytosis ;it has no role in management of PCT

Which of the following hemolytic anemias can be classified as extra corpuscular? A: elliptocytosis B: paroxysmal nocturnal hemoglobinuria C: pyruvate kinase deficiency D: sickle cell anemia E: TTP

E hemolytic anemias may be either intra or extra corpuscular. In intra-, the patients RBCs have an abnormally short lifespan due to an intrinsic RBC factor. In extra-, the RBC has a short life span due to a non-intrinsic RBC factor. TTP is an acquired disorder where red and PLT destruction occurs not because of defects in their cell lines, but rather a result of microangiopathy leading to destructive shear forces on the cell. Other ssx include fever, AMS, and less commonly, renal impairment. All cases of hemolysis in c/w thrombocytopenia should be rapidly ruled out for TTP by evaluation of PBS for schistocytes as plasmapheresis is life-saving. Other causes include hypersplenism, AIHA, DIC, and other microangiopathic hemolytic anemias. The other 4 listed all refer to some defect of the RBC itself which leads to hemolysis. Elliptocytosis is a membranopathy that leads to varying degrees of destruction of the RBC in the RES. SCD is a congenital Hgbopathy classified by recurrent pain crises and numerous longterm sequelae that is due to well defined B globing mutation. PK def is a rare disease of the glycolytic pathway that causes hemolytic anemia. PNH is a form of acquired hemolysis due to an intrinsic abnormality of the RBC. It is often causes of thrombosis and cytopenias. BM failure is a feared association with PNH.

81 YO M admitted for AMS. He was found at home, confused and lethargic, by his son. His PMH is significant for mets prostatic cancer. The patient's meds include periodic intramuscular goserelin injections. On exam, he is afebrile. BP is 110/50, HR 110. He is lethargic and minimally responsive to sternal rub. He has bitemporal wasting, and his mucous membranes are dry. On neuro exam, he is obtunded. The patient has an intact gag reflex and withdraws to pain in all 4 extremities. Rectal tone is normal. Labs are significant for Cr 4.2, Ca 12.4, and Alb 2.6. All of these are appropriate initial management except: A: normal saline B: pamidronate C: furosemide when the patient is euvolemic D: calcitonin E: dexamethasone

E hypercalcemia is a common oncologic cx of mets. SSX include lethargy, confusion, AMS, fatigue, polyuria, and constipation. Regardless of the underlying disease, the treatment is similar. These patients are often dehydrated, as hyperCa can cause nephrogenic DI, and are often unable to take po fluids. Therefore, the primary management entails reestablishment of euvolemia. Often hyperCa will resolve with hydration alone. Bisphosphonates are another mainstay of therapy as they stabilize osteoclast resorption of Ca from the bone. However, their effects take 1-2 days to manifest. Care must be taken in renal insufficiency as rapid administration of pamidronate may exacerbate renal failure. Once euvolemia is achieved, furosemide may be given to increase calciuresis. Nasal or SUBQ calcitonin further aids the shift of calcium out of the intravascular space. Glucocorticoids may be useful in patients with lymphoid malignancies as the mechanism of hypercalcemia in those conditions is often 2/2 excess hydroxylation of Vitamin D. However, in this patient with prostate cancer, dexamethasone will have little effect on the calcium level and may exacerbate the AMS.

A patient with ALL is admitted with respiratory distress and CP. The patient reports 1 day of SOB not a/w cough. There have been no sick contacts, and before the onset of respiratory ssx, the patient only recalls fatigue. A CXR shows faint diffuse interstitial infiltrates w/o pulmonary edema. The cardiac silhouette is normal. An ABG shows PaO2 54 and Pulse Ox 97% on RA. A CO level is wnl. All of the following labs are expected in this patient, except: A: bcr-abl mutation B: blasts count > 100K C: elevated LDH D: increased blood viscosity E: methemoglobinemia

E hyperleukocytosis is a potentially fatal cx of acute leukemia when the blast count > 100K. Cx of this syndrome are mediated by hyper viscosity, tumor aggregates causing slow blood flow, and invasion of the primitive leukemic cells, which causes hemorrhage. The brain and lungs are MC involved. The pulmonary syndrome may leads to respiratory distress and progressive respiratory failure. CXR may show either alveolar or interstitial infiltrates. A common finding in patients with markedly elevated immature WBC counts is low PaO2 on ABG with normal pulse ox. This may actually be due to pseudohypoxemia, because WBC rapidly consume plasma oxygen during the delay between collecting arterial blood and measuring oxygen tension, causing a spuriously low measured O2 tension. Placing the ABG immediately on ice will prevent the pseudohypoxemia. The bcr-abl fusion mutation is found in up to 25% of patients with ALL. In addition, as tumor cells lyse, LDH levels can rise rapidly. Methemoglobinemia is usually due to exposure to oxidizing agents such as ABX or local anesthetics. Respiratory ssx may develop when methemoglobin levels are >10-15% (depending on the Hgb concentration). Typically arterial PaO2 is normal and measured SaO2 is inappropriately reduced because pulse ox is inaccurate with high levels of methemoglobin

All of the following are late cx of BMT preparative regimens, except: A: growth retardation B: azoospermia C: hypothyroidism D: cataracts E: dementia

E in addition to chronic GVHD, there are late cx of BMT which result from CTx and RTx preparations. Children may experienced decreased growth velocity and delay in the development of secondary sex characteristics. HRT may be necessary. Gonadal dysfunction is common. Men frequently become azoospermic, and women develop ovarian failure. Patients who received total body irradiation are at risk for cataract formation and thyroid dysfunction. Although cognitive dysfunction may occur in the peritransplant period for many reasons, there is no definitive evidence that dementia occurs at an increased frequency.

22 YO F comes to the ED with 12 hours of SOB. The ssx began towards the end of a long car ride home from college. She has no PMH and her only meds is OCP. She smokes occasionally but the frequency has increased recently because of examinations. On PE, she is afebrile with RR 22, BP 120/80, HR 110, Sa 92% on RA. The rest of PE is wnl. CXR and CBC are wnl. Her serum pregnancy test is negative. Which is indicated? A: check D-dimer and, if normal, d/c with NSAIDs B: check D dimer, and if normal, obtain LE US C: check D-dimer, and if abnormal, treat for DVT/PE D: check D-dimer, and if abnormal, obtain contrast multislice CT of chest E: obtain contrast multislice CT of chest If this patient develops a R-PE and is started on LMWH and Warfarin, what is the goal of the INR and duration of the treatment?

E the clinical prob of PE can be delineated into likely vs unlikely using the clinical decision rule shown below. A normal D-dimer combined with unlikely clinical prob of PE identifies patients who do not need further testing or AC. Those with either a likely clinical prob (>4) or abnormal D-dimer (with unlikely clinical prob) require an imaging test to r/o PE. Currently the most attractive imaging is multislice CT. It is accurate and, if normal, safely r/o PE. This patient has a clinical prob score of 4.5 because her resting tachycardia and lack of alternative diagnosis at least as likely as PE. Therefore, there is no indication for measuring D-dimer, and she should proceed to CT. If this cannot be performed expeditiously, she should receive one dose of LMWH while waiting. The goal of treatment with Vitamin K antagonists, including Warfarin, is to maintain the INR 2-3, with a goal of 2.5. High intensity treatment is not more effective and has a bleeding risk. Lower intensity treatment is less effective, with a similar bleeding risk. Generally, this patient should be treated for 6 months

46 YO F presents with new onset ascites and severe ABD pain: a hepatic Doppler reveals hepatic vein thrombosis. She also reports tea colored urine on occasion, particularly in the morning, as well as recurrent worsening ABD pain. On exam, she is found to have undetectable serum haptoglobin, elevated LDH, hemoglobinuria, and elevated reticulocyte count. PBS shows no schistocytes. What is the diagnosis? A: adenocarcinoma of the ovary B: antiphospholipid syndrome C: aplastic anemia D: Factor V def E: PNH

E the combination of intravascular hemolysis (hemoglobinuria) and thrombosis in an unusual location (particularly in proximity to the abdominal viscera) should prompt a search for PNH. PNH results from an acquired mutation in SC resulting in the loss of glycosylphosphatidylinositol-linked cell surface membrane proteins in a clone of granulocytes. Dx is made by flow cytometry of CD55 or CD59 expression on the granulocytes. The Ham or sucrose lysis tests are no longer routinely performed. Clones of deficient cells are often detected in patients with aplastic anemia. Adenocarcinomas are strongly a/w thrombosis (Trousseau's) and may cause ascites, but hemolysis without microangiopathic hemolytic anemia makes this less likely. Other causes of hyper coagulable state such as those listed should be examined if the eval for PNH is negative.

You are managing a patient with suspected DIC. The patient has ESLD awaiting liver transplant and was recently in the ICU with E coli bacterial peritonitis. You suspect DIC based on a new UGIB in the setting of oozing from venipuncture sites. PLT 43, INR 2.5, Hgb 6, and D-dimer is elevated to 4.5. What is the best way to distinguish between new onset DIC and chronic liver disease? A: blood cx B: elevated fibrinogen degradation products C: prolonged aPTT D: reduced PLT count E: serial labs

E the ddx between DIC and severe liver disease is challenging. Both entities may manifest with similar labs: elevated fibrinogen degradation products, prolonged aPTT and PT, anemia and thrombocytopenia. When suspecting DIC, these tests should be repeated over a period of 6-8 hours because abnormalities may change dramatically in patients with severe DIC. In contrast, these tests should not fluctuate as much in patients with severe liver disease. Bacterial sepsis with positive blood cx is a common cause of DIC but is not diagnostic

50 YO female presents to clinic for evaluation of elevated PLT count. The latest CBC is WBC 7, HCT 34, and PLT 600. All of the following are common causes of thrombocytosis, except: A: Fe def anemia B: essential thrombocytosis C: CML D: myelodysplasia E: pernicious anemia

E thrombocytosis may be "primary" or "secondary"; essential thrombocytosis is a myeloproliferative disorder that involves a multipoint hematopoietic stem cell. Unfortunately, there is no clonal marker that can reliably distinguish it from MC nonclonal, reactive forms of thrombocytosis. Therefore, the diagnosis is one of exclusion. Common causes of secondary thrombocytosis include infection, inflammatory, malignancy, Fe def, hemorrhage, and post-op states. Other myeloproliferatives like CML and myelofibrosis, may result in thrombocytosis. Similarly, myelodysplastic syndromes, particularly 5q, may cause thrombocytosis. Pernicious anemia caused by b12 def does not. However, correction of B12 or folate def may cause a "rebound" thrombocytosis. Similarly, cessation of chronic ethanol use may also cause a rebound thrombocytosis

all of the following are vitamin K dependent coagulation factors, except: A: factor X B: factor VII C: protein C D: protein S E: factor VIII

E vitamin K is a fat-soluble vitamin that plays an essential role in hemostasis. It is absorbed in the SI and stored in the liver. It serves as a cofactor in the enzymatic carboxylation of glutamic acid residues on prothrombin-complex proteins. The three major causes of vitamin K deficiency are poor dietary intake, intestinal malabsorption, and liver disease. The prothrombin complex proteins (2, 7, 9, 10, C + S) all decrease with vitamin K deficiency. Factor VII and Protein C have the shortest half life of these and therefore decrease first. Therefore, Vitamin K deficiency manifests with prolongation of the PT first. With severe deficiency, the aPTT will be prolonged as well. Factor VIII is not influenced by Vitamin K

Which of these correctly describes characteristics of stem cells? A: ability to ddx into a variety of mature cell types B: capacity for self renewal C: generate, maintain, and repair tissue D: A and C E: A and B F: All of these

F all peripheral blood cells and some cells in peripheral tissue are derived from hematopoietic stem cells. When they are irreversibly damaged, as in severe radiation exposure, an individual cannot survive longer than a few weeks. The two cardinal features of SC are the ability to ddx into a variety of mature cell types and the capacity for self renewal. The ability to ddx allows SC to participate in the maintenance and repair of tissues. In addition, the capacity for self renewal assures an ongoing supply of SC to continually maintain adequate tissue function. These characteristics are the basis for the growing excitement regarding their use for a vast array of medical conditions, including (but not limited to) diabetes, spinal cord injury, cardiomyopathy, heme disorders, and enzyme deficiencies

the evaluation in a newly diagnosed case of ALL should routinely include all of the following, except: A: BM biopsy B: cell-surface phenotyping C: complete metabolic panel D: cytogenetic testing E: lumbar puncture F: plasma viscosity

F viscosity testing is typically reserved for cases of multiple myeloma where paraproteins (particularly IgM) can lead to vascular slugging and subsequent tissue ischemia. ALL can lead to end-organ damage in kidney and liver, therefore routine chemistry tests are indicated. A LP must be performed in cases of newly diagnosed ALL to r/o spread to the CNS. BM biopsy reveals the degree of marrow infiltration and is often necessary for classification of the tumor. Immunologic cell-surface marker testing often indicates the cell lineage involved and the type of tumor, information that is often impossible to discern from morphology alone. Cytogenetic testing provides key prognostic information on the disease natural history

The MC inherited prothrombotic disorder is:

activated protein C resistance Factor V Leiden mutation refers to a point mutation in the factor V gene (arginine to glutamine at P506). This makes the molecule resistant to degradation by activated protein C. This disorder alone may account for up to 25% of inherited prothrombotic states, making it the MC of these disorders. Heterozygosity for this mutation increases an individuals lifetime risk of DVT sevenfold. A homozygote has a 20-fold increase risk of thrombosis. Prothrombin gene mutation is probably the 2nd MC condition that causes hyper coagulability. Antithrombin, protein C, and protein S deficiencies are more rare. Antithrombin complexes with activated coagulation proteins and blocks their biologic activity. Deficiency in anthrombin therefore promotes prolonged activity of coagulation proteins, resulting in thrombosis. Similarly, protein C and S are involved in the proteolysis of factors Va and VIIIa, which shuts off fibrin formation. Because protein C and S are dependent on vitamin K for carboxylation, administration of warfarin A.C. may lower the levels of protein C and S more quickly relative to II, VII, IX, and X, thereby promoting coagulation. Patient with Protein C deficiency may develop warfarin-related skin necrosis

58 YO F presents to ED c/o jaundice. She first noticed yellowish discoloration of her skin about 3 days ago. It has become progressively worse since that time. In a/w the development of jaundice, she also has noticed clay-colored stools and pruritus. There has been no associated ABD pain, fever, chills, or night sweats. She has PMH of ETOH abuse, but has been abstinent for the past 10 years. She has no known hx of cirrhosis. On PE, she is afebrile with normal VS. She is jaundiced. The BS are wnl. The ABD is soft and nontender. There is no distention. The liver span is 12 cm to percussion and is palpable at the R-costal margin. The spleen tip is not palpable. LFTs reveal AST 122, ALT 168, ALP 483, bilirubin t/d 22/19. On RUQ US, the gallbladder cannot be visualized, and there is dilatation of the intrahepatic bile ducts but not the CBD. What is the diagnosis?

cholangiocarcinoma this patient is presenting with painless jaundice and acholic stools. On RUQ US, the gallbladder cannot be visualized, suggesting collapse of the GB. In addition, there is dilatation of the intrahepatic bile ducts, but not the CBD, suggesting a tumor at the bifurcation of the CBD. This type of cholangiocarcinoma is called a Klatskin tumor. The incidence appears to be increasing. In general, the cause of most cholangiocarcinoma is unknown, but there is increased risk in PSC, liver flukes, ETOH abuse, and any cause of chronic biliary injury. Cholangiocarcinoma typically presents as painless jaundice. Imaging usually shows dilatation of the bile ducts, and the extent of the dilatation depends on site of obstruction. Dx is usually made during ERCP, which defines the biliary tree and allows a biopsy to be taken. Hilar cholangiocarcinoma is resectable in about 30% of patients, and the mean survival is 24 months. Cholecystitis is typically a/w fever, chills, and ABD pain. The degree of jaundice would not be expected to be as high as seen in this patient. GB cancer should present with a gallbladder mass, rather than collapsed GB, and chronic RUQ pain is usually present. HCC may be a/w painless jaundice but it is not a/w dilatation of intrahepatic bile ducts and the marked elevation in ALP. Malignancy at the head of the pancreas may present in a similar fashion but should not result in collapse of the GB. In addition, the CBD should be markedly dilated.

73 YO M presents with 3 months of increasing back pain. He localizes the pain to the lumbar spine and states that it is worse at night while he is lying in bed. It is improved during the day with mobilization. PMH is notable only for HTN and remote cigarette smoking. PE is wnl. Labs are notable for +ALP. A lumbar XR shows a lytic lesion in the L3 vertebra. Which malignancy is likely?

Non-small cell lung cancer bone pain resulting from mets may be difficult to distinguish from degenerative disease, osteoporosis, or disk disease in the elderly. Generally, these patients present with insidious worsening localized pain without fevers or signs of infection. In contrast to pain related to disk disease, the pain of mets is worse when the patient is lying down or at night. Neurologic ssx related to mets constitute an emergency. Lung, breast, and prostate cancers account for approximately 80% of bone mets. Thyroid, renal, lymphoma, and bladder may also mets to bone. Mets can be lytic or blastic. Most cancers cause a combination of both, although prostate cancer is predominantly blastic. Either lesion may cause hypercalcemia, although lytic lesions more commonly do this. Lytic lesions are best detected with plain XR. Blastic lesions are predominant on radionuclide bone scans. Treatment and prognosis depend on the underlying malignancy. Bisphosphonates may reduce hypercalcemia, relieve pain, and limit resorption.


Kaugnay na mga set ng pag-aaral

Modeling With Quadratic Functions (quiz)

View Set

Psychology: A History (Structuralism, Functionalism and Psychoanalysis)

View Set

Ch. 8 Relevant Costs for Short-Term Decisions

View Set

16-18 apush test questions from quizzes

View Set

Network Security 1.0 Modules 15-17

View Set